AMA | April 2024

Welcome to the April 2024 Ask Me Anything episode of Mindscape! These monthly excursions are funded by Patreon supporters (who are also the ones asking the questions). We take questions asked by Patreons, whittle them down to a more manageable number -- based primarily on whether I have anything interesting to say about them, not whether the questions themselves are good -- and sometimes group them together if they are about a similar topic. Enjoy!

AMA

Support Mindscape on Patreon.

AMA questions April 2024

Jeremy Dittman
I really enjoyed your episode with Matt Strassler last month. At one point in your discussion, you ask Matt for his definition of a field at its most fundamental level and he answered that it’s something extended throughout the cosmos with the feature that if you change it in one place, it has effects in other places in the future. This seems to place causal structure as the rock-bottom essence of what a field is. Do you agree with this definition? Is causality fundamental in this sense or something better considered as emergent?

Ned Grady
Your catch phrase 'Sorry about that' has creeped it's way into my speech. I hope you're happy. Where did you pick it up?

naveed alam
what intimidates you? is there a person that might give you pause on a chance meeting and with whom you would very much want to but struggle to make conversation?

Shubhendu Harsh
Hypothetically if one of your colleagues, a brilliant human being and an even better theoretical physicist, leaves a note saying, "All the evidence points to a single conclusion: Physics has never existed, and will never exist. I know what I'm doing is irresponsible. But I have no choice." and disappears. What would you do?
Excerpt from one of my favorite SF novels, Three Body Problem which is being made as a NF series.

Jeffrey Segall
I was struck by the sequence of your conversation with Sahar with the possibility that social media might provide a universal coupling constant coordinating spins in an Ising transition model and then your solo episode discussing Geoffrey West and the idea that communication between people has been driving innovation. Was placing these two episodes one after another purely coincidental or did your conversation with Sahar inspire you to do that solo episode?

Jeff B
If we conceptualize the relationship "x is emergent from y" as a directed graph, how can we know whether this graph has closed loops? In other words, how can we know whether there is a unique foundational structure from which all other ways of viewing the world are emergent, versus there being a number of equivalent conceptualizations that can all be thought of as emerging from each other?

Paul Cousin
Your reflections video on the Sahar Heydari Fard episode reminded me of a question I have been pondering for some time. It seems that the human brain is capable of capturing structures of arbitrary scales in its connectivity / electrical activity (i.e. knowledge about particle physics, astrophysics, or for that matter society). Could this sort of scale-freeness be related to the critical brain hypothesis? More generally, it seems that our universe has to allow structure to transcend scale for anything interesting to happen. Has this been shown to be a property of the laws of physics?

Don McKenzie
Thanks much for the great explanation of parallel transport on pp. 205-208 of Biggest Ideas 1. Can you explain which two indices (vectors) in the Riemann curvature tensor are contracted for the Einstein equation, and wny?

Tripp Denison
Recently I heard Nicolas Gisin's idea about how indeterminacy exists in classical physics. He avers that if classical systems evolve according to truly chaotic equations, and the physical properties involved in those equations can have real number values which are irrational and aperiodic and therefore require an infinite amount of information to fully specify, then those chaotic dynamics would have to involve infinite information density which is associated with black holes or is otherwise impossible. He concludes that classical physical properties cannot have real number values but are uncertain and become more specified through interaction, allowing for indeterminacy in the evolution of classical systems. Will you please share your thoughts about this argument?

Tim Converse
Why does the Pauli Exclusion Principle apply to fermions and not to bosons?

*************************************
Mick
PRIORITY QUESTION. I enjoyed your recent podcast with Robert Sapolsky and have just read Free Agents by Kevin J Mitchell. These debates seem to place determinism as a critical concept. If you’ve dealt with this elsewhere, please point me in the right direction, otherwise how do you think about determinism (generally, and in this debate)?

Justin Wolcott
Can you elaborate on the difference between compatibilism and hard determinism?
This is how it sounds to me:
Hard Determinist: the universe is fundamentally deterministic. This is at odds with the concept of freewill.
Compatibilists: the universe is fundamentally deterministic. This is at odds with the concept of freewill - from the god's eye view. Nobody is god. Therefore, for all intents and purposes its ok to act as if freewill is real.
************************************

Corey Leander
What are you thoughts on the Einstein Cartan theory of gravity and its avoidance of singularities? Why isn’t this theory taken more seriously?

Simon Carter
Will you be coming to the UK this year as part of your Biggest Ideas in.The Universe 2 book tour? And have you chosen a topic to present yet?

Callan
Priority Question:
My question is, what would you say to someone who is struggling with anxiety, or angst over the implications of naturalism? I'm not so scared of dying, oddly enough. But I do find it difficult to accept determinism, for some reason it just bothers me. I suppose it undermines my sense of control over my life. I really look up to you as someone who's helped guide me in life through your work, and you seem to be an optimistic, easy-going guy. So I'm just wondering, did you ever struggle with angst like I'm feeling much?

Danielle Lanz
Can you explain the preferred basis problem? Do you think it poses an actual challenge to MWI?

Scott Stambach
Priority question: There is a philosophical position you may be familiar with called anti-natalism. The idea is that human procreation is much more ethically tricky than people appreciate. Anti-natalists like David Benatar argue that every human life experiences mild to severe amounts of suffering and that no human life consents to their own existence. Looking at the history of humanity, so full of profound suffering for certain individuals and groups, I have to admit that it is hard for me not to be sympathetic to these arguments. I’m wondering what your thoughts are on anti-natalism and what points you might make in debate with someone who holds this position.

Bob Ritchie
Regarding the Fermi paradox, I keep reading that self-replicating Von Neumann probes would be easy to produce. Really? Mining asteroids and creating "free" new probes with sophisticated propulsion, navigation and communications systems, saturating the galaxy, and waiting thousands of years for reports? Your thoughts?

Charles E. Grant
I'm fascinated by the idea that space-time emerges from entanglement. But I'm confused. In the laboratory we can maximally entangle two particles in a Bell state and we then can separate those two particles by as great an interval as we want (within the light cone). We could also do this with two particles that are completely unentangled. Naively this would seem to indicate that entanglement and space-time separation are independent.

TheGreatDeceiver
Simple question this month…When is a good time to give up on a dream? Have you ever had to give up on a dream?
Here in Canada, amongst several other crises, there is a housing crisis, even more acute than in the U.S. Being a bit late to the home ownership game I now find myself being forced to give up the dream of ever owning a little house. As a single, middle class earner it is simply not possible. The average home here costs about 700k. There are millions in the same position, it’s just kinda sad. But here we are.

*********************************
Avaneesh Narla
Do you think that spin glasses are appropriate models for real-life complex systems (such as societies and economies)?

Herbert Berkowitz
In your recent solo episode you mentioned “econophysics.” It is an apparent attempt by physicists to apply the principles of physics to the field of economics. Do physicists really think that they have something to bring to that table? What does the behavior of particles have to do with the behavior of people?
*********************************

Shawn Khanna
Given the surprising findings in quantum biology over the past few years, do you think there may be salient quantum effects in the brain? Are quantum effects likely to play an essential role in perception, cognition, or consciousness?

Frank Schulz
In Episode 110 with Neil Johnson on Complexity, Conflict, and Infodemiology, you were discussing the possibility and feasibility of controlling e.g. radical fringe groups and the spread of misinformation in information networks in the modern world (social media, etc.).
I was a bit shocked that during conversation, neither of you were realizing how being able to control fringe ideas and boiling up of social dynamics in these networks can be abused by e.g. a totalitarian regime. Shouldn't we be more careful (or at least aware) when doing this kind of research what happens when it falls into the "wrong" hands?

Nathan
How has the Francis Scott Key bridge collapse affected you or those you know well? Hope all is well

Scott D
My AMA question is as follows: What are your thoughts regarding Eric Verlinde's idea's on gravity as an emergent force are correct for our universe?

Eduard Sackinger
Towards the end of your conversation with Matt Strassler you mentioned that there are three languages to talk about gravity: forces, curved
spacetime, and a condensate of gravitons. Could you please explain to what extent these languages are equivalent? Is it possible, for example, to describe a black hole with gravitons in flat spacetime?

Michael
I know you are a 76ers fan, but, since it also seems you're a fan of basketball generally, I am wondering what your thoughts are about Victor Wembanyama.

Donald Wilcox
How would you describe space time in terms of gravitons?

Igor Kopylov
If Laplace's demon wanted to find an emergent higher level theory of the system it's simulating (e.g. planets orbiting the sun from an atom by atom simulation of the solar system) is there an efficient way for it to do that? On one hand, we've found those theories without being demons, on the other hand, considering every possible coarse-graning seems like a much bigger problem than even simulating everything at the lowest level.

population_thinking
I have a question about your discussion with Lara Buchak on risk and rationality. Can you give us some more intuition about the difference between a risk-averse utility function and Buchak's "risk-avoidance"?

Owe
Why do matter/antimatter collisions release an enormous amount of energy? If two inverse sound waves collide, the sound is nullified (which I believe is the basis for noise canceling headphones) - why does this not extend to particles, given that they are also waves?

Richard Kashdan
In the film Interstellar, the crew is walking around on a planet whose gravity is so strong that 7 years on Earth pass for every hour they spend on the planet. So how come they walk around normally without being affected by this amazing gravity?

Connor
Here's a question I had after listening to your latest episode with Claudia de Rham. As the universe expands, the furthest stuff we can see gets progressively red shifted until we can no longer see it. Similarly, if you watch an object fall into a black hole, you’ll see it get progressively red shifted until it fades completely. Is there more to this similarity than just coincidence? Are there deeper connections between the cosmic horizon and an event horizon?

Leo Behe
I'm interested in the parallels between lossy data compression in computer science and scientific models of physical systems. In both cases, we’re designing a sort of rule or algorithm that allows us to discard a large amount of information about a system while retaining the parts of the system that are “meaningful” to us. It seems that if we were Laplace’s demon and we had all the information about a physical system at all points in space and time, we wouldn't need to describe it with physical laws. Do you consider this a potentially useful comparison or is it too broad or vague to really be saying much?

Dave Grundgeiger
In your recent solo podcast, "The Coming Transition in How Humanity Lives," you expressed doubt that we could be the same type of consciousness after uploading our minds into computer simulations. You said, "If you take the information that is in your brain and encode it in some computer chip, you have removed its connection to your body. And what we think about as human beings is inextricably intertwined with their bodies." Why do you think (if you do) that this objection won't be overcome by faithfully simulating the experience of having a body? That doesn't seem different from simulating any other part of the world, which is already presumed in the premise.

lemmy101
PRIORITY QUESTION: Using my priority question for a TV show recommendation of all things! I absolutely loved your solo ep discussing time travel stories, and was happy to hear you were a fan of Timecrimes too. In describing the type of time travel stories that you like which mirrors my own, my mind was screaming DARK the entire time. Dark is a German show on Netflix and the most interesting, well executed and complex time travel story I've ever seen, if you or anyone listening hasn't seen should definitely give it a go!

Leeroy
Re: your thoughts about "designer babies" in the recent solo podcast; are there any specific issues or scenarios that you're particularly concerned about?
As someone preoccupied with the history of disability politics, medicine and eugenics, I think this issue is likely more complex and ethically ambiguous than we imagine.

Ken Wolfe
Both your recent solo episode on the Transition in How Humanity Lives and the previous discussion with Sahar Heydari Fard on Complexity, Justice and Social Dynamics prompted me to step back and think of the apparent implicit assumption that there is an agreed upon goal. I suppose preventing the extinction of our species is a base goal, though some might even question that. I would be skeptical about any general reference to human flourishing since libertarians and various flavors of communitarians might have radically different and incpmpatible ideas on what that should look like. Besides just survival is there anything you think we can all agree on, or will further divergent hetrogeneous social contracts be inevitable?

anonymous
Have you found evidence for aliens or God? I would like to be anonymous.

Corey G. Riker
i was thinking about symmetries and conservation laws the other day, and i became curious as to weather this relation could be thought of as stemming from orthogonalities in hilbert space. disclaimer, most of what i know about physics is from popular science books and podcasts. so i asked chatgpt , and it applauded my insightfulness! it felt good, but i am well aware that gpt is not perfect at identifying true statements. so give it to me straight sean , did i realize something true?

Curious Dennis
PRIORITY QUESTION
I have a nagging question about your Biggest Ideas in the Universe. Given that religion has been virtually ubiquitous within all of our human societies, I think that Meaning or the search for Meaning might be as essential to the human organism as, let's say vitamin C. One of the things that comes to mind when looking at the night sky is: What is the meaning of all this? Don't you think that a cosmologist, like yourself, with a strong background in philosophy, should include Meaning as one of the Biggest Ideas in the Universe?

Nikola ivanov
The AdS/CFT correspondence doesn’t reflect our observable universe not only on the AdS side of the equation but also on the CFT side. The Standard Model as a QFT doesn’t seem to be scale invariant and it doesn’t seem to exhibit conformal invariance. However, my impression is that scientific efforts seem to be focused much more on finding dS space correspondence to CFT and less on replacing the CFT part of the equivalence with the QFT of the Standard Model. Is my impression correct and if it is why is this the case?

Chris Guenther
Could physical symmetries (like gauge symmetries) be akin to redundancies in information? As in, if a file has redundancies in the data (like an ASCII text file), that means there is room for compression to provide a more compact representation. Could physical symmetries point to a more compact physical representation that does not have symmetries? In how you and others talk about symmetries, they seem to be the progenitor of many physical phenomena, like how photons have zero mass.

Amy Ferguson
Do you often remember your dreams, and have any been especially memorable or led to a lucid dreaming experience?

Allan Lubel
Love your podcasts, it's great to have a healthy addiction! And I've listened again to some of your early ones and noticed that you seem to laugh more often in them that in the more recent ones. If I'm right about this, do you feel that since you were new to podcasting that the laughs were due to a little nervousness (which is totally understandable) or rather is it that in the process of getting older you have become more serious?

Matthew Lounsbury
"PRIORITY QUESTION"
While looking up to the sky I have been tantalized with the thought that I can only see stars because those little photons have actually traveled all this way and entered and messed with some rhodopsin in my eye; that it arrived with enough energy to change my being in some meaningful if absolutely tiny way. This feels to me significant. Since time for the photon slows to nothing and space shrinks before it: is it fair to say that there is an actual point in the universe ( the photons point of view?) Where distant Andromeda and the cells in my eye are connected at one place and time? That the photon leaves there and enters me here with no distance to cross, no time needed to move across it, a.k.a no movement needed?
Are we in some sense touching the stars, or they us? That feels too poetic to believe?

Kent Durham
Your solo podcast indicated that we would reach a point to where firms would extract our wealth at maximum efficiency. A counterpoint might be that firms need to continue to compete against others and that this competition would lead to further enhancing the goods and services that we consume. That enhancement might preserve our standard of living ad finitum. Do you have an interesting answer to this observation?

Alex Thew
I really enjoyed your discussion with Matt Strassler. There are many confusing ideas about physics that are probably owed to a limitation of language. For that matter, the idea of referring to a variety of fundamental things (i.e. protons, electrons, photons, etc.) as “particles” has always confused me. Should we be thinking of these things as spheres? Solids? Or are “particles” intended to be a heuristic to conceptualize a three-dimensional point in space(time)? How do physicists think about particles as opposed to, say, a high school physics student?

**************************************
Kyle Stephens
In your recent solo episode you outline both optimistic and pessimistic visions for how humanity will live in the future. What can we as average people do to increase the probability of the optimistic scenario coming to fruition?

Nick Gall
I thoroughly enjoyed your solo podcast on 'The Coming Transition in How Humanity Lives'. You outline optimistic and pessimistic scenarios for the next cultural equilibrium, but what about a "not better, not worse, just (trans formatively) different" scenario? All our previous cultural transitions seem to better fit this "just different" characterization: new sources of suffering emerge along with new forms of joy. Has any cultural era ever looked back and thought their era lived up to optimistic expectations or down to pessimistic ones?

Philip Grant
Concluding your solo episode on the technological singularity, you rightly say that "we need to collectively decide to avoid the pessimistic possible outcomes and work for the more optimistic ones." How optimistic are you that humanity will find ways of making truly global collective decisions, rather than stronger nations imposing their interests on weaker ones?
*************************

Reid Atherton
I'm intrigued when you've said that modern levels of productivity imply that we can meet the basic needs of all people. But I'm curious what the feasible method is for actually doing this, leaving behind a spherical cow understanding? For instance centralized economic control has been worse at incorporating all the necessary information into managing surpluses and shortages, compared to market pricing.

Ilia Lvov
You often close your pop-sci explanations with a line akin, “If you want to understand more, you need to look at the equations”. I understand where this comes from. Yet I can’t help but make a mental connection to a particularly difficult course in Macroeconomics I had in my undergrad studies. That course was so difficult precisely because the professors made us go *beyond solving the equations*. The equations were always the easiest part by far. The difficult parts were:
(1) to give a causal interpretation (“change in X results in change in Y results in change in Z, etc. etc. hence we have a new equilibrium”);
(2) to give a graphic interpretation (to graphically show the movement of the fundamental economic curves, such as the Liquidity and Money curve in the IS-LM Model).
Is there a similar risk in Physics that focusing solely on the equations would strip away some of the understanding? And what other modes of understanding beyond the equations are similarly helpful to achieve a deeper understanding in the Physics studies?

Kevin James
My understanding is that it takes negative gravitational energy to create a wormhole. It appears that dark energy behaves sort of like negative gravitational energy. I know you'll say yes it's possible (but I'm asking anyway), if we could hypothetically harness dark energy as best we know about it now, could we create wormholes?

Henry Jacobs
In the bonus reflection video for Sahar Heydari Fard you were struck with the notion of "cultural revolutions as critical phenomena", where an idea can spread like wild-fire if the network is scale free. This reminds me of stuff in the air in the mid 90s (often at the Santa Fe institute) related to "the sandpile game". Not a specific questions here, but do you have any thoughts on the sandpile game and the research around it? I get the impression that it fizzled, but maybe not.

************************
Sandro Stucki
In the episode with Matt Strassler, you both agreed that particles in QFT are not really point particles but modes or excitations in a quantum field. But Feynman diagrams look like they describe the interactions of point particles. What am I missing?

Claudio
The physics of an incandescent bulb seems pretty straightforward, electrons running on a thin tungsten wire experience resistance, generating photons and heat according to simple equations. But how does it work in terms of QFT? Can you explain how, in the space occupied by the wire, excitations in the electron field turn into excitations in the electromagnetic field, but only under precise conditions (e.g., wire properties, current, etc)?
**************************

edward crump
If string theory is correct, in some form, could the strings purported to constitute the fabric of space pass through the nucleus of an atom and even between two quarks?

Matthew Atkins
How important is the cosmological principle to modern cosmology? Do we have good reason to believe it or is it just a convenient assumption? What are the implications if it turns out to be false?

Rad Antonov
In your view, does the Everettian interpretation of quantum mechanics resolve apparent fine tuning puzzles, such as CP symmetry in QCD and the values of certain constants, by allowing for the existence of an infinite number of combinations of coupling constants?

Paul Hess
We visited Johns Hopkins yesterday as part of my son’s college search and stopped by Papermoon Diner based on your recent mention of it. He says Hopkins is elevated on his list in part because your role there is a sign of openness towards interdisciplinary thinking, and he likes the annual lecture series you’ve mentioned helping to organize there (Dennet & West so far). So take this as a small sign of the positive feedback loop that science communication and outreach efforts can have. He’ll be applying to schools with a focus on Classics but finds appeal in Philosophy, Physics and Math so is focusing on medium+ schools that have strengths in all of those areas. This is not a question just a thank you, and I’m equally happy if you comment or don’t comment on this among the flood of actual questions.

yehonatan Peretz
If we find through learning about quantum gravity that the spacetime curvature doesn’t go to infinity inside a black hole would that do any violence to the statement that no light can escape it?

Qubit
If we are faced with a very hard decision, do you think the resulting outcome corresponds to two branches with roughly equal weight or is the overwhelming weight still concentrated on one of the two branches? The latter case seems to speak against your statements about many worlds being comforting because you know yourself to have chosen differently in another universe.

William Briggs
I am a 3rd year medical student considering a career in academic pathology. What advice do you have to someone considering academia? How should I try to find my area of research and set myself up to be successful?

Orrin Cummins
Does 2+2=4 in every branch of the multiverse, and what does that tell us about the underlying structure of Many Worlds?

David Wright
There is an ongoing debate regarding the ethics of generative AI producing images, music, and literature without crediting the original human contributors. My intuition is that published material represents a kind of collective unconscious that AI draws on to create its outputs. This could lead to a feedback loop where AI shapes our collective experience and influences our future. Over time, AI could produce documentaries, literature, and cultural beliefs that alter our understanding of history. Is this a serious risk to our civilization?

Ken
What is the answer to Elisabeth’s question to Descartes?: “Given that the soul of a human being is only a thinking substance, how can it affect the bodily spirits, in order to bring about voluntary actions?”

Kyle M. Kabasares
At what point did you subscribe to (and hit the ‘like’ button) the Many-World’s interpretation of quantum mechanics? I’m interested in knowing if it was before, during, or after your first formal introduction to quantum mechanics in the classroom.

David Maxwell
I love your solo episodes. They're always on engaging topics and elucidate that topic in a way few other sources do, even when you're not the subject matter expert. Your latest one on the coming transition was brilliant and timely. So why don't you do more?
I could see reasons including additional effort, having a finite number of things to opine on for a few hours, solos being less popular with occasional listeners, or just that you started this to interview others, not give monologues. But I adore them and I suspect I'm not alone.

C Handley
I hope Ariel and Caliban can help me with this marital issue. When my wife and I got married we made up some silly little vows to go along with the more serious ones. At the time, she had one cat (Gatsby) that I understood would be part of the family. I like cats, but not as much as my wife. One of her vows was, "I understand we will never have more than one cat." We now have three. Stella, Penny, and Pete. I argue that since the vow was broken at two different times, I have two vow breaks in my back pocket for future use. She argues that a single vow was broken, period, so that's just one allowable vow break. Can physics provide some insight into who is right here? Also, am I warranted to have a relatively high credence that when I break my vows, she will understand? 🙂

 

0:00:00.1 Sean Carroll: Hello, everyone, and welcome to the April 2024 Ask Me Anything edition of the Mindscape podcast. I'm your host, Sean Carroll. The big news around here is that because it is April, it will soon be May, and May 14th will be the release date of my next book, Quanta and Fields, The Biggest Ideas in the Universe, Volume 2. I don't want to say too much about the book because I'll probably just do a solo podcast related to it somehow at some point very, very soon. I'll take requests for anything that people specifically want to know about quantum field theory, which is most of what the book is about. You know the basic idea of the Biggest Ideas series, which is that I'm gonna be teaching physics to people who are non-physicists, so it's not a textbook or anything like that, but I do the equations. So I'm gonna show you equations, I'm gonna try to explain what they are, and this book is action-packed. We're gonna do quantum field theory as well as quantum mechanics, and we're gonna skip over some of the mathematical details because there are just too many in quantum field theory, but you will meet the bosons and fermions of the standard model, you will meet Feynman diagrams and Lagrangians and renormalization.

0:01:11.7 SC: You will think about spontaneous symmetry breaking and confinement and the spin statistics theorem and how it all comes together to do atoms and molecules and so forth. The reason why I am mentioning all of this is because, given that it is a bit more than a month before the release date, I have been doing the audiobook. I have been reading the audiobook. As you know, I read my own audiobooks. I didn't read the first two, from Eternity to Here and then The Particle at the End of the Universe. Other people read them. And, you know, given the specific nature of this kind of material, it's just better if I read it. So I've been doing it since then, especially this stuff with all the equations and everything. And let me tell you, it's taken it out of me doing the audiobook. That's basically what I'm here to say. Both trying to do the work of translating the text into audio form because I have to explain in words the figures, the equations, etc. It's not just a completely straightforward reading of the text. And then, you know, talking for many, many hours. My voice is kind of shot. You can probably tell that. Yet here I am. I got to do the AMA.

0:02:20.6 SC: And I look forward to doing the AMA every month. So I'm happy to do it. But it might be a little scratchy here and there. Fair warning. That's all I'm trying to tell you there. And given all that, there's no reason to delay. So let's go.

[music]

0:02:52.1 SC: Jeremy Dittman says, I really enjoyed your episode with Matt Strassler last month. At one point in your discussion, you asked Matt for his definition of a field at its most fundamental level. And he answered that it's something extended throughout the cosmos with the feature that, if you change it in one place, it has effects in other places in the future. This seems to place causal structure as the rock bottom essence of what a field is. Do you agree with this definition? Is causality fundamental in this sense, or something better considered as emergent? Well, I think there's a couple things going on here. One is, what do you mean by causality? And the other, of course, is the explicit question, what do you mean by a field? You know, I have frequently talked about cause and effect relationships in the macroscopic world as something that is emergent, that is a higher level thing.

0:03:38.8 SC: What I mean by that is that the fundamental laws of physics, whether it's Newtonian mechanics or the Schrodinger equation or quantum field theory or whatever, are Putting aside some very minor questions about the quantum measurement problem, but the basic dynamical equations of physics do not distinguish between past and future. They are reversible. They are time reversal invariant, two closely related concepts but not quite the same. Anyway, the point is that if your fundamental dynamics are unchanged by switching past to future, then you can't have any kind of idea that there are things called causes which always come temporally before things called effects, right? And yet that notion is super useful in our actual world. And I think that it's pretty clear that that's because our everyday notion of cause and effect arises from a universe with a strong arrow of time at a higher level of macroscopic emergent behavior. You know, the arrow of time, as we've often talked about, is only macroscopic.

0:04:47.8 SC: It doesn't exist there at the microscopic level. It's because of entropy increasing over time in a coarse-grained view of the universe. And the same thing is true for cause-and-effect relationships. The reason why I'm saying this is because that is not what most particle physicists mean by causality. They have borrowed the word causality to mean something other than cause-and-effect relationships, which is fine, but it can be confusing when they use it in front of other people. All particle physicists mean is things travel slower than the speed of light or at the speed of light, right? So you poke the universe in one place and at the fundamental level, you could either have ripples of things happening moving toward the future or ripples of things having happened coming into you from the past. Equally well, you could solve the equations of motion both ways. Those of you who are physics majors know that this is an explicit thing. When you study electromagnetism, there's sort of different kinds of solutions to the equations with past and future boundary conditions, etc.

0:05:56.1 SC: So to move on to what Matt was really talking about vis-a-vis the field concept, I would not put causality up there, even in this sense, when I really talk about what a field is. But, you know, it's perfectly okay for him to do so. I think the point was that in that discussion, he was less interested in the very broadest idea of what a field might be and more interested in the actual definition of a field as we use it in real-world quantum field theory in the standard model of particle physics. So it's not that the causal structure of the world and the light cones and the inability to go faster than the speed of light is necessarily part of the definition of a field. It's that since we have those things in the real world, those things play a really important role in how fields actually behave in the world. For example, for a counterexample in some sense, my favorite example of the first time when the field concept really came into physics was with Pierre-Simon Laplace, one of our favorite people here at Mindscape, one of our favorite mathematical physical thinkers.

0:07:05.8 SC: Among other things that Laplace did besides his demonology and so forth, he was the one who showed how you could take Newtonian gravity, right, the famous inverse square law of gravity and think about it in terms of a field theory. The field in that case is the gravitational potential field. There is an equation, Laplace's equation it is called, for how the field responds to mass and energy, if you want, in the universe. And it is precisely mathematically equivalent to good old Newtonian gravity. It is slightly more convenient for some purposes because you don't need to think of individual parts of the gravitational sources as points with masses, you can have a distributed source, you can have energy density spread out throughout the universe. But really, it's just a matter of convenience. The two formalisms are precisely equivalent. The reason why I'm mentioning this is because Laplace's gravitational potential field version of Newtonian gravity is Newtonian. It is not relativistic. There are no light cones. There is no speed at which influences propagate across the universe. But it is a field theory.

0:08:19.5 SC: It's just a field theory where if I move a mass right here, that instantly changes the value of the field all throughout the universe. There is no upper limit on the propagation speed. But that's not the real world either. So it just matters. It's just a matter of whether or not you are taking the actual features of the real world as really important when you define what this concept means to you. Ned Grady says, your catchphrase, sorry about that, has creeped its way into my speech. I hope you're happy. Where did you pick it up? I honestly can say that I've never realized that that was a catchphrase. Sorry about that. Yeah. But probably it is. One doesn't recognize very often one's own frequent phrases. It's really not a catchphrase. I don't think anyone would recognize it. But it could be something that I lean on quite often. I have zero idea where I picked that up. But, you know, maybe it's just guilt talking. You know, maybe it's just I want to keep the audience happy.

0:09:25.0 SC: Sometimes I can't do it. I have to apologize every so often. So I'm gonna claim to have made that up all by myself. Navid Alam says, what intimidates you? Is there a person that might give you pause on a chance meeting and with whom you would very much want to but struggle to make conversation? Well. I don't think there's a person like that, no. There would have been when I was younger, but I think I've grown past that because I've been able to live a life that is, you know, sufficiently fortunate that I've met a lot of incredibly impressive, intimidating people. And certainly, believe me, I've been intimidated many, many times. But at some point, you meet people who are super duper good at their job, super duper talented, super duper smart, or whatever, accomplished. And they're just people at the end of the day. I've heard very smart people say and do very dumb things, say and do silly things and very warm and human things, etc. There's really no reason to be intimidated by human beings. I do think that, again, people are super impressive. That's a different thing.

0:10:37.2 SC: I just think that, you know, I heard recently a piano recital by Michael Hirsch who is a Professor, among other things, he's a composer and performer, mostly a composer. He's at Johns Hopkins. He's at the Peabody School, which is part of Johns Hopkins. And it's, his ability to play the piano is just superhuman, and the things that he composes, he said afterward, you know, that people were asking him, and he said, I don't think anyone else can play these pieces, basically. I've heard people try, and it's not quite the same. He has a really extraordinary ability to play the piano, and that's endlessly impressive to me. I'm not intimidated by him, personally. He seemed like a nice guy, honestly. But There are certain many, many dimensions of human accomplishment at which people are far, far surpassing anything I could ever imagine. But I think that my attitude toward those people is one of admiration and being impressed, but not really one of being intimidated.

0:11:40.6 SC: Supendu Harsh says, hypothetically, if one of your colleagues, a brilliant human being and an even better theoretical physicist, leaves a note saying, all the evidence points to a single conclusion. Physics has never existed and will never exist. I know what I'm doing is irresponsible, but I have no choice. And then they disappear. What would you do? This is an excerpt from one of my favorite science fiction novels, The Three-Body Problem, which is being made as a Netflix series. I think that it would depend a lot on details about this person, etc. But I think that the chances that I would take such a note at face value as truth are very, very small indeed. In the space of all possible ways that such a note could come to exist, many more plausible ones involving that one brilliant theoretical physicist being wrong or having a breakdown of some sort. There's many more plausible versions of that story than there are of physics not existing. So not to say that there aren't events that would cause me to believe that kind of dramatic claim, but the evidence would have to be enormously stronger than that.

0:12:54.5 SC: That's all I will say about that. Jeffrey Segal says, I was struck by the sequence of your conversation with Sahar Haidari Fard with the possibility that social media might provide a universal coupling constant coordinating spins in an Ising transition model. And then also your solo episode discussing Jeffrey West and the idea that communication between people has been driving innovation. Was placing these two episodes one after another purely coincidental or did your conversation with Sahar inspire you to do that solo episode? It was mostly coincidental, I think. The thing that really inspired me to do it was listening to Jeffrey's talk that he gave as the lecturer here at Johns Hopkins. But let me be very honest. I think that people sometimes exaggerate the extent to which you can pinpoint knowledge in your brain or even ideas in your mind as originating from some particular source, okay? In other words, people say, well, when did you first think of this?

0:14:04.4 SC: Or what caused you to have this idea, right? And almost always, I think, in the real world, for the vast majority of thoughts that I have, I don't know is the answer. Because many, many things happen. I mean, I do the podcast every week and that's only a small fraction of all the things I do and the people I talk to every week, right? So many things are going on in my mind, including many things I'm reading and books I'm planning to write and have read myself and so forth, and things develop slowly and gradually, and there's little shifts of emphases and reshuffling of cards that are already there and so forth. You know, the stuff that I talked with Sahar about, is stuff that I've thought about in different ways for a long time. I mean, even here on the podcast, when we talk with Caelan O'Connor, for example, we talked about similar things. I had a whole course that I taught to Johns Hopkins on the physics of democracy, which will someday become a book and so on. So there's no question that talking to her absolutely gave me good ideas and helped me think about things, but I can't relate exactly the words in that conversation to exactly what I talked about in the podcast episode.

0:15:12.5 SC: So I just think I'm not giving you the actual answer to your question, but I think that it's a mistake to try to be too... To try to relate things that you read too directly with things that you think, because so many things that you think are being caused by subconscious things or ways of mixing pre-existing ideas that you're not even aware of at the surface. Jeff B. Says, if we conceptualize the relationship X is emergent from Y as a directed graph, how can we know whether this graph has closed loops? In other words, how can we know whether there is a unique foundational structure from which all other ways of viewing the world are emergent versus there being a number of equivalent conceptualization that can all be thought as emerging from each other? That's very good. I'm sort of suspicious that Jeff B. Is someone that was just at a workshop I was at at the Santa Fe Institute because we were talking about exactly that idea. David Krakauer, former Mindscape guest, and also David Wolpert, another SFI professor, organized a workshop on investigating reality where they were proposing to think about how we think about reality in terms of an Ouroboros.

0:16:32.9 SC: The Ouroboros is the snake that eats its own tail, the serpent that eats its own tail, right? And they were saying, you know, you can think of physics as giving rise to chemistry and chemistry to biology and biology to the mind and thinking. And then the mind invents mathematics, and mathematics is the foundation of physics. So you've circled around. So rather than being a foundational structure, it is more like a circle, just like that. Maybe you've not been involved with that, Jeff, but that was certainly exactly the idea that we had. And a couple of us at the meeting raised our hands and said, no, it's really not like that at all, because those relationships are not really quite the same relationship. I mean, physics depends on math, but so does biology, so does chemistry. And the relationship between math and physics is just not the same as the relationship between physics and chemistry. It's a different kind of relationship.

0:17:31.8 SC: At a more specific down-to-earth technical level, the emergence relation is irreversible. It is one way because it is an information-compressing relationship. The way that you go from physics to higher-order things is to throw away information, right? If you're Boltzmann, you're defining entropy as the logarithm of the number of microstates in a macrostate, you need to have macrostates, and you need to not care which specific microstate you're in. So it's a one-way street, this emergence map, as I think about it. Of course, other people have defined emergence differently, so they might tell you a different story, but that would be my take on it. Paul Cousin said, your reflections video in the Sahar Haidari Fard episode reminded me of a question I have been pondering for some time. Let me mention, let me pause for a second to mention that Patreon supporters, who are of course the ones asking these Ask Me Anything questions. Actually, I didn't even mention that, did I?

0:18:35.9 SC: By the way, the Ask Me Anything questions are asked by Patreon supporters, and you could be one of those Patreon supporters. I'm supposed to give this sales pitch every time. Just go to www.patreon.com/SeanMCarroll, and you can sign up to be a supporter of Mindscape, dollar an episode, or however much more you want to give. You get ad-free versions of the episodes, as well as the ability to ask the AMA questions. And I've been experimenting with this idea where after every regular podcast, not the AMAs, but regular podcasts where I'm interviewing somebody, I do a little five-minute reflection video where I just talk about my immediate in-the-moment impressions of how the conversation just went, you know, ideas that come to mind after having that talk. So if you listened to the podcast I did with Sahar, and this was just referenced in a previous question, but there is a particular idea in physics that I had a conviction for a long time must be relevant to thinking about society and the emergent behavior of human beings.

0:19:50.3 SC: And that is, of course, the idea of a phase transition, right? It's sometimes talked about in the social context as a tipping point. When you go from being in one phase to being in another phase, like water turning to ice or to water vapor or what have you. Fine, that's pretty easy and that's pretty straightforward. But there is a specific quantitative feature of physics-based phase transitions, which is critical or scale-free behavior. That is to say, something is going on at all different length scales. If you have the Ising model or water boiling or whatever, when you are exactly at that phase transition point, there's activity at very, very small scales and at very, very large scales. And on either side of the phase transition, you will generally get either nothing but large-scale activity or nothing but small-scale activity. It's really at that critical phase transition point where the activity is going on at all scales.

0:20:51.5 SC: And so Sahar mentioned that this is a feature of networks of information flow in human societies that are able to enact... Enact? Is that the right word? To generate social change, that there are both sort of tightly knit people and also more weak bonds to broader people throughout the community more generally. If you have only one but not the other, if you have only weak connections to a large number of people or strong connections to a small number, you're less able to get real social change happening than if you have both. So I thought that was an interesting connection. Anyway, I talked about that in the reflection video for Sahar's episode. So Paul continues. He says, it seems that the human brain is capable of capturing structures of arbitrary scales and its connectivity and electrical activity, I.e. Knowledge about particle physics, astrophysics, or for that matter, society. Could this sort of scale freeness be related to the critical brain hypothesis? More generally, it seems that our universe has to allow structure to transcend the scale for anything interesting to happen.

0:22:03.9 SC: Has this been shown to be a property of the laws of physics? Well, so this is referring to an idea that the structure of the brain is also critical, scale-free. I mean, not even the structure of the brain, the structure of the thinking brain, I should say. The idea here is that when your brain is conscious and you're thinking and going about your day, there are connections that are very local. You know, one neuron's talking to the next neuron, but there's also wider fluctuations, like a whole area of your brain lights up for some reason or another. And there's a claim that is slightly controversial, as I understand it. I am not an expert. But the claim is that there's sort of power law behavior, that in a well-functioning conscious brain, there is structure at all scales in a way that looks just like a phase transition criticality. But it's not, of course, a phase transition. There's nothing changing from one state to another.

0:23:00.1 SC: It's just operating at that critical point, okay? And interestingly, part of the people who claim this is that, you know, when you're asleep or you're unconscious or you're under anesthetics, this scale-free behavior is not there. You have the sort of local connections, one neuron talking to another, but you don't also have the wider correlations that are featured in consciousness. Take that for what you will. Again, I don't even know if it's true. This is the kind of thing that people argue about in neuroscience. So I take the first part of Paul's question to be... I'm not sure what the first part is. He says, could this sort of scale freeness, I.e. Yeah, could this sort of scale freeness be related to the critical brain hypothesis? I think the sort of scale freeness is the critical brain hypothesis, as I understand it. But I don't understand it very, very well. So maybe there is something else going on that I'm just not aware of. And then more generally, it seems that our universe has to allow structure to transcend scale for anything interesting to happen.

0:24:08.7 SC: Well, this is a good question. This is something that is, again, a frequent. Source of study and controversy in the study of complex systems, the ubiquity of scale-free behavior and power laws in particular. By power laws, what we mean is if you have some way of characterizing how often something happens, then as the scale gets bigger and bigger, the frequency with which that thing happens declines as a power of the size, the size to some power, okay? And if you plot that on a log-log scale, it will look like a straight line. And people like Nicholas Taleb have written a lot about this in finance markets, black swans, things like that. Has this shown to be a property of the laws of physics? No, not as far as anyone knows. So I would encourage you to go back to the episode we did with Nigel Goldenfeld, who's one of the world's physics experts on these phenomena. So it's the nomenclature, the way that we talk about it is a little confusing, I admit. And I struggled with this when I was in my physics classes, we talk about a phase transition, and as soon as we talk about that, we give you examples like water melting or boiling, right?

0:25:29.4 SC: And other kinds of transitions from one phase to another. That's very, makes perfect sense. But all of those are sort of happening over time, right? Here is the water. Oh, it has boiled. Now it is water vapor. So you tend to think of a phase transition as a dynamical process, but then when you get more sophisticated at it, you just think of the point at which the phase transition would happen if it were gonna be happening, but you're sort of sticking it there. You're not letting the system deviate from this point where the phase transition happens. And that's where you get these statistics of power law behavior and so forth. And so the question I asked Nigel, and to which I still don't personally know the answer is, it seems as if that critical tipping point would be like a very special point, right? Certainly, if you're increasing the temperature of water to make it boil, there's only one temperature at which water boils. There's a lot of temperatures at which it's liquid or vapor, and only one that is in between. So it would seem unnatural to have a system be stuck at precisely that value.

0:26:41.4 SC: And yet, in nature, we get all sorts of scale-free behavior. Especially in biology, I will have to say. There's certain non-biological examples. Turbulence is the most obvious example. If you have a correctly viscous fluid and drag a stick through it and make some big waves, they will sort of dissolve into smaller and smaller waves on smaller and smaller scales, and it will look like a fractal. A fractal is scale-free behavior. And so that's pretty robust and has nothing to do with biology. But in biology, scale-free behavior is all over, whether it's trees or the organs inside your body, your circulatory system, respiratory system, etc. These look very scale-free in an obvious way, and arguably, likewise, your brain. So it seems to me that in those biological cases anyway, things are not very robust. Like biological organisms can die, right? They don't reassemble themselves if they're blown to bits.

0:27:47.6 SC: They are being kept at a particular configuration and structure and dynamics by taking in food and fuel from their environment and using it to self-repair and stay alive and stay in a certain configuration. But more generally, why do... But it's not only biological things. I mentioned turbulence but I don't know if you know Zipf's Law. If you talk about the frequency with which different words appear in a text, and you talk about the least, the most likely words occur a certain frequency, and then as you get down in the histogram, you get less and less likely. Guess what? It's a power law, right? It's nothing biological about it or anything like that. So this is a mystery to me. I mean, I think this is something where... I don't want to say it's a mystery like at a super profound level. I just think that it is perfectly plausible in my mind that different examples of critical behavior and power laws happen for completely different reasons. And therefore, I'm a little reluctant to think about some grand unified theory of all of them. But it is completely possible to me that I just don't understand it very well.

0:29:07.4 SC: And so I'm not giving you the right answer. That's why I'm saying it's a mystery to me, not necessarily to the world, but just to me personally. Don McKenzie says, Thanks so much for the great explanation of parallel transport on pages 205 to 208 of The Biggest Ideas in the Universe, Volume 1, Space-Time in Motion. Well, you're welcome, Don. Can you explain which two indices of the Riemann curvature tensor are contracted for the Einstein equation and why? So, yeah, for those of you who are fans of volume 1 of the biggest ideas in the universe, you know, the capstone achievement by the end of that book was Einstein's equation for the metric tensor in general relativity responding to energy and momentum in the universe. And that is intermediated via the Riemann curvature tensor, which is a mathematical object you can construct from the metric and from this idea of parallel transport, using the metric to define a way of transporting vectors around some space or space-time. And again, in that book, I tell you what a tensor is. One way of thinking what a tensor is, it's kind of like a generalization of a matrix.

0:30:21.4 SC: If you think of a vector as a column of numbers. V vector is VX, VY, VZ, I can just write the components of that vector as V sub I, right? Where I could be X, Y, or Z. And then if I have more than one index on my object, so instead of VI, I have SIJ, then I have a little matrix. Then I have a little 3 by 3 matrix if I goes from 1-3. In relativity, these indices are generally given by Greek letters and they go from 0-3, where zero is time, because these are the dimensions of spacetime. So you have something like t mu nu, the energy momentum tensor of general relativity, and mu and nu are indices that go over the values zero, one, two, three. Time, space, space, space. One dimension of time, three dimensions of space. And once you have that idea, there's no problem having more than two indices in your object. And tensors are kinds of these objects. And look, the real experts in tensor analysis out there are pulling out their hair in frustration right now because I am not giving you the correct high-level definition in terms of multilinear maps.

0:31:42.7 SC: But you can buy my textbook to find all that. Believe me, it is in there. Anyway. It turns out that in general relativity, or forget about general relativity, just the idea of Riemannian geometry, it turns out that you have this metric tensor, which is a two-index tensor, g mu nu, which tells you the distances along curves from which you can get volumes and areas and all those other things. But the curvature of that metric tensor is described by a different tensor, the Riemann curvature tensor, and that has four indices. R lambda rho mu nu, for example. So that's a problem because if you want to invent equations of motion for gravity, you would like to relate the curvature of spacetime to the energy inside. And as I just mentioned, the energy momentum tensor, t mu nu, only has two indices. And you cannot relate in any obvious way a four-index tensor to a two-index tensor. So various mathematical niceties are involved, but the answer is to what we call contract the Riemann tensor.

0:32:53.3 SC: So if you have r lambda rho mu nu, four indices, you can track the first index and the third one of those four. And what I mean by that is instead of r lambda rho mu nu, four different indices, I take r0 rho0 mu. So I plug in the value zero both for the first and the third index. Plus r1 rho1 mu, plus r2 rho2 mu, plus r3 rho 3mu. So I add up those four values where that first index and the third index take on the same values. In fact, it doesn't really matter which indices you contract over each other because there's a lot of symmetries of the Riemann tensor, and you could get equivalent formulations in various different ways. And what you do at the end of the day by getting this is something called the Ritchie curvature tensor. It's basically a subset of the Riemann tensor. It is a particular kind of formulation, a particular kind of characterization, I suppose is the right word, of what the curvature is doing. The full 100% of the curvature is covered by the Riemann tensor.

0:34:11.4 SC: The Ritchie tensor is only telling you some of the curvature. And then what happens is you can use that Ritchie tensor to invent something called the Einstein tensor, which you set equal to the energy momentum tensor, and that's Einstein's equation. There you go. Now, so I think I have explained which two indices are contracted. But by the way, Don, if you want that level of specificity, you really should just get a real general relativity textbook, not the biggest ideas book. You're all ready... You're ready to dive in and do it correctly. And why, you ask? You know, there's so many different ways to answer that question. Why are these two indices the ones that are contracted? Before I attempt to answer it, let me, well, I'll try to answer it. My favorite way of thinking about where Einstein's equation comes from is the action principle, which actually was pioneered by David Hilbert in this context. The action is the integral of the Lagrangian over time or the integral of the Lagrange density over space-time.

0:35:20.3 SC: And there's a very natural Lagrange density to use for general relativity, which is just the contraction of the Ritchie Tensor called the curvature scalar, R. And basically, because you can't have any indices in a Lagrange density, I know that many of you listening have no idea what I'm talking about right now. Sorry about that, but maybe you can read the book. The point is that if you have that very, very obvious choice of Lagrange density, and then what you're supposed to do to get equations of motion is take the derivative of that Lagrange density with respect to the field that you're taking as your fundamental variable, in this case the metric tensor field. And in fact, there's a subtlety there I'm not telling you about because that's not quite the Lagrange density. Lagrange density is that times the square root of the derivative of the metric to make the volume element come out okay. Clearly, as I'm giving this explanation, it is becoming clear to me that I'm not gonna be able to give this explanation in an understandable way.

0:36:24.3 SC: But the punchline would be, if you understood anything of what I was saying right now, the punchline would be that starting with a very natural choice of action principle, And going through the variational calculus needed to turn that action principle into equations of motion, this particular set of subset of the Riemann curvature tensor is what pops out mathematically and is gonna be set equal to the energy momentum tensor. Everything that I said right there is much more understandable than I made it sound. So I encourage any of you who are even mildly interested to check it out. It's actually quite worth understanding. But the other thing I wanted to say is that once that happens. Something interesting does actually happen when this is, again, I'm gonna be a slight exaggeration, slight simplification, but I hope some of the truth gets through. What you've done by taking the whole Riemann curvature tensor, which has many, many components, and contracting it to get the Ritchie tensor, so now you're looking at a subset of it.

0:37:30.7 SC: And you're setting that subset of it proportional to the energy momentum tensor. And what that means is that once you have that relationship, once you have general relativity in Einstein's equation, there is sort of a physical meaning to the equations for both the Ritchie tensor, a la Einstein, and also the equations for all of the Riemann tensor that is not in the Ritchie tensor. That is something called the Weyl tensor, not because it's Weyl, but after Hermann Weyl, W-E-Y-L. And roughly speaking, the Ritchie tensor part controls Newtonian gravity, like the gravity in the solar system, the gravity that you get when you have some object and is creating gravity in a gravitational field. And the Weyl tensor part affects gravitational waves, characterizes gravitational waves. So something that is brand new in Einstein's equation compared to Newtonian gravity is that there can be a wave of gravity traveling at the speed of light. This goes back to what we talked about with Laplace earlier. In Laplace's gravitational potential field version of Newtonian gravity, there's no gravitational waves. But in Einstein's there is because Space-time's metric has a life of its own. So even if I have no idea, or even if there's no stuff in the universe, let me put it that way, even if the universe is completely empty.

0:38:56.0 SC: I still haven't told you exactly what the gravitational field is, 'cause there can be independently propagating gravitational waves. And those are described using the Weyl tensor, and Einstein's equation ultimately governs how to describe them, even though it's not explicitly there, so it's a whole complicated fun story. But that's, I think that maybe what is getting at, what is motivating your question there, like how can I get a successful theory of the curvature of space-time if I only have an equation that talks about some of it?

0:39:28.6 SC: The answer is that there are relations between the different parts, and indeed, even the rest of the Riemann curvature tensor is very tightly constrained by the equation you write down for general relativity. Trip Dennison says, recently I heard Niklas Giesen's idea about how indeterminacy exists in classical physics. He averts that if the classical systems evolve according to truly chaotic equations, and the physical properties involved in those equations can have real number values which are irrational and aperiodic, and therefore require an infinite amount of information to fully specify, then those chaotic dynamics would have to involve infinite information density, which is associated with black holes, or is otherwise impossible.

0:40:13.9 SC: He concludes that classical physical properties cannot have real number values, but are uncertain and become more specified through interaction, allowing for indeterminacy in the evolution of classical systems. Will you please share your thoughts about this argument?

0:40:27.1 SC: Well, I'll confess I don't know the argument very well, so I'm relying on your description of it, but in your description, it seems completely unconvincing. Let me put it that way. Just so everyone knows, when we talk about truly chaotic equations, chaotic is different than indeterministic. A classic example of chaos is the double pendulum. So you take like a pendulum, just hang a pendulum in a very ordinary way, but then from the bottom, from the weight of that pendulum, you hang another pendulum. This is something you can then swing and watch what happens. It's perfectly deterministic. It's perfectly classical in the classical approximation anyway, Newton's laws suffice to predict it, but the motion is chaotic. And what that means is that small variations in the initial conditions lead to large variations in the ultimate evolution.

0:41:19.2 SC: That's completely compatible with classical mechanics. So this argument that Tripp has rehearsed here hinges on this idea that because in classical mechanics, continuous values are involved, like the parameter for the angle of the oscillator or the position of a particle or whatever is a real number, and therefore to specify a real number exactly takes an infinite amount of information, right, 'cause there's an infinite number of real numbers between zero and one. Then it says the chaotic dynamics would have to involve infinite information density, which is associated with black holes. That's just out of nowhere. I have no idea what that is about.

0:42:05.7 SC: Infinite energy density, maybe, or even just very large energy densities are associated with black holes. I mean, if anything, infinite information density is not associated with black holes. It's sort of the opposite of the conclusion being drawn here because the Bekenstein-Hawking entropy of a black hole is a finite number. If you think of the Bekenstein-Hawking entropy as the entanglement entropy of degrees of freedom inside the black hole and outside, and think that the black hole is maximum entropy, both of which are perfectly reasonable assumptions, then that's telling you that there's only a finite amount of information in the black hole. But anyway, who cares about black holes or anything like that? You can just do classical mechanics.

0:42:52.2 SC: You can just do classical mechanics and say, all right, I have some matter here. Will a black hole form? And the prediction is unambiguous, like typically, no, black holes don't always form in the process of chaotic systems doing their thing. This might be some sort of fun motivational line of reasoning to get people to think that quantum mechanics is in some sense better defined than classical mechanics. And I think that's actually a reasonable argument. Quantum mechanics being based on the Schrodinger equation is linear. The Schrodinger equation is a perfectly linear equation. It doesn't have any chaotic behavior in terms of what the wave function does. In fact, for a while there, there was debates about how can there be chaotic behavior in the world if the Schrodinger equation is linear and the world is fundamentally quantum mechanical.

0:43:44.3 SC: The answer, of course, is that there is something called the classical limit. So you are seeing in the classical limit chaotic behavior, even though the underlying behavior of the wave function is perfectly linear and non-chaotic. And so that's interesting, but it's not truly attention or anything like that. So anyway, whenever arguments like this are presented, the real answer is, who cares? Classical physics is not right. You know, John Norton famously came up with the idea of Norton's dome, which is a much more convincing argument. The classical Newtonian mechanics is not deterministic in the way that is usually stated, only because there's sort of a set of measure zero, set of initial data, where the future evolution is not fully specified by the equations. That's also possible. But again, it's not that interesting to physicists because the world is not fundamentally classical and these issues just don't arise in quantum mechanics.

0:44:48.9 SC: Tim Converse says, why does the Pauli exclusion principle apply to fermions and not to bosons? Well, I mean, these why questions are always going to depend on what level of explanation you're looking for here. As you will find out in my upcoming book, Quanta and Fields, there are two types of fields in relativistic quantum field theories, at least in four spacetime dimensions. There's more interesting things that can happen in lower dimensions. But in three plus one dimensions, you have bosons and fermions. And bosons are fields that basically can pile up on top of each other. Excitations in the fields can have the individual excitations, which you and I would identify as particles, can occupy the same quantum states. They can redundantly be in the same quantum states. That's why you get big classical force fields like gravity and electromagnetism.

0:45:43.7 SC: Whereas fermion fields are those that cannot be piled on top of each other. They take up space. That's why matter tends to be made of fermion fields like electrons and protons and neutrons, which are made of quarks, also fermions, because they take up space. They give heft to you and the tables and chairs around you, et cetera. Now it is also true, and this is not necessary, but it is a theorem that it is true, that bosons have integer spins, 0, 1, 2, 3, et cetera. And fermions have half integer spins, one half, three halves, five halves. I suppose they should say it is necessary, but it's not part of the definition. It is something you prove about the relationship between spin and statistics. Statistics are, in this case, the word referring to whether the particles are bosons or fermions.

0:46:40.6 SC: And so the definition of bosons is not particles with integer spins. That is a feature of them that you can derive, something called the spin statistics theorem. And I talk about the spin statistics theorem in the book. I do not prove it. I gesture at a proof. In fact, the status of the proof is a little bit dicey, as it turns out. You know, I'm pretty sure it has been proven, but I'm pretty sure that very few people understand the proof. Everyone points to this famous book by Streeter and Whiteman, I think, PCT, spins, statistics, and all that, like a little mathematical physics book from the 1960s that purports to prove the theorem. But there's a lot of ways you can sort of make the theorem seem reasonable. But it turns out those ways are not completely mathematically legitimate. So that's what everyone does, like every quantum field theory book, et cetera, including my book. And basically, all these arguments come down to the following thing.

0:47:42.2 SC: When you think about the wave function, the quantum wave function of a particle, you know, it has some shape in space, for example. And what happens if you rotate all of space by 360 degrees? Usually, you think of rotating things of 360 degrees as not doing anything, do anything, right? You know, it's just returning you to where you started. But for fermion fields, when you rotate by... I'm sorry, I shouldn't say that. For spin 1 half fields, like the electron, when you rotate by 360 degrees, you actually pick up a minus sign, an overall multiplicative minus sign in the wave function. Of course, that leads all of the physics completely invariant, because things depend on the wave function squared. So picking up a minus sign is no big deal. For bosons, a spin 0 boson is not changed by a rotation. A spin 1 boson is left invariant by 360 degrees. But a spin 1 half particle, like an electron, you need to rotate by 4 pi, by 720 degrees, okay?

0:48:49.5 SC: A simple once around rotation picks up a minus sign. Those are features of single individual particles with spin. Here is a separate feature of fermions, that if I take two fermions that are identical to each other, and I exchange them with each other, I take particle A, particle B, I switch particle A with particle B, the overall wave function of the two-particle system picks up a minus sign. And for bosons, they do not. Why is that true? Well, you can wave your hands a little bit. That's the part where, you know, people, it depends on exactly what assumptions you're willing to take. But the key insight is that these are in some sense the same minus sign. The minus sign that you pick up when you take a single particle and rotate its wave function by 360 degrees is the same minus sign as if you take two particles and exchange them.

0:49:45.1 SC: And the fact that when you take two particles and exchange them, you pick up a minus sign, that's where the Pauli exclusion principle comes from. Because if the two particles were in the same quantum state, then exchanging them doesn't do anything. And 'cause they're the same, right? But then you exchange them and you pick up a minus sign that is incompatible with not doing anything. So that's the origin of the Pauli exclusion principle. You can see from that explanation that there's a lot of hand-waving going on. It's certainly not mathematically rigorous. That is the typical level of understanding that your working particle physicist has about these things.

0:50:20.3 SC: Okay, I'm going to group two questions together. The first one is from Mick and is a priority question. Remember, priority questions are things that you as a Patreon supporter get to ask once in your life and I promise to do my best to answer them. So Mick's question is, I enjoyed your recent podcast with Robert Sapolsky. It's not that recent anymore, but okay, I did do a podcast with Sapolsky. And I've just read Free Agents by Kevin Mitchell. These debates seem to place determinism as a critical concept. If you've dealt with this elsewhere, please point me in the right direction. Otherwise, how do you think about determinism and in general about this debate? And then Justin Wolcott says, can you elaborate on the difference between compatibilism and hard determinism?

0:51:03.4 SC: This is how it sounds to me. A hard determinist will say the universe is fundamentally deterministic. This is at odds with the concept of free will. Compatibilists say the universe is fundamentally deterministic. This is at odds with the concept of free will from the God's eye view. Nobody is God. Therefore, for all intents and purposes, it's okay to act as if free will is real. So I'm glad that Mick labeled this a priority question because I would not have answered it if it had not been labeled a priority question. Good call, Mick. And only because I've answered this question before. And I've also mentioned before that I have run out of things to say about free will. So I do my best, but I will happily say them again if you want.

0:51:46.8 SC: The point is, there's many points here. Number one, as a compatibilist... Actually, maybe I should answer Justin's question first so I can clarify what I mean by compatibilism. So Justin is saying that a hard determinist will say determinism is at odds with the concept of free will, whereas a compatibilist will say a deterministic universe is at odds with the concept of free will, but only from the God's eye view. So no, that is not how I would gloss compatibilism. I would gloss it exactly as I always do, which is to say free will is compatible with determinism. To me, free will is a higher-level emergent concept. Free will is just as real as tables and chairs and baseball and puppies, okay? None of which are to be found in the standard model of particle physics. But the whole point of higher-level emergent theories is that we throw away information.

0:52:41.2 SC: We describe the system using incomplete information. I can describe the table in front of me without knowing the precise quantum microstate of the table, of all the atoms, all the quantum fields that go into it. Nevertheless, despite not knowing that, the idea of the table plays a very important role in the causal structure of the world around me. You can tell me there's a table there and I instantly know that there are certain features associated with it. I can put my coffee cup on it and it will hold it. Things like that, okay?

0:53:11.7 SC: It is a miraculous feature of emergence that you don't need all of the information in order to describe the world at a higher level. It is a feature of human beings at our best emergent level of description that we can't predict what they're going to do. It is true that if you knew all of the quantum mechanical microstates of the system, then you could make a prediction for what they're going to do. And it would only be probabilistic because measurements are probabilistic, et cetera. But I don't. And I won't. And most importantly, the theory doesn't. The theory that we have for describing tables and chairs and human beings doesn't include the complete microstate of the universe. So the fact that there exists a different theory, the core theory or whatever, the God's-eye-view theory, that does make such predictions is irrelevant to the capacities of the higher-level emergent theory.

0:54:12.2 SC: In the higher-level emergent theory, we don't have such an ability. Or even I can put it even more precisely, in the higher-level emergent theory, part of the conceptual framework, just like tables and chairs, is the agency of human beings. It's not mystical. It's not weird. It's not incompatible with the laws of physics. But we attribute to human beings predispositions and preferences and rational thought and cognition. And again, as I always say in these debates, literally everybody does. Like, there's no people out there who say, I'm a hard determinist and I have never spoken about human beings as if they make choices. Those people don't exist. Okay? Everyone does this. The compatibilists are just saying, admit it. Admit that you do this. Face up to the fact that this is the correct way of describing human beings.

0:55:05.1 SC: So it's not that it's okay for all intents and purposes to act as if free will is real. Free will is real at that emergent level as real as tables and chairs. Okay, so that's compatibilism. Now, I should point out, so this goes back to Mick's question, the universe is not deterministic, right? There might be some underlying deterministic theory, like many worlds or Bohmian mechanics or whatever, that is ultimately describing what happens in the wider universe, including both what we can see and what we can't. But our branch of the wave function, our observable world, is not deterministic because of quantum mechanics. So as you can tell from the previous elaboration of compatibilism, who cares? I truly don't care. About whether or not microphysics is deterministic, it wouldn't matter for the point of view of compatibilism. Compatibilism rests on understanding the universe at a higher emergent level.

0:56:05.0 SC: For some reason, there are people who don't like free will and who bang on about determinism, which is both wrong and irrelevant. I mean, what, you know, double double, as you would say in basketball terms. So there you go. If you want to be pointed in the right direction, Mick, I might have said Mitch by accident, sorry. Mick, if you want to be pointed in the right direction, I do talk about this in a blog post called Free Will is as Real as Baseball. But that was pretty old. I have a slightly more up-to-date discussion in The Big Picture, the book that I wrote, where you can read at least a little bit about it. I don't spend too much time because I just said everything I got to say. Corey Leander says, what are your thoughts on the Einstein-Cartan theory of gravity and its avoidance of singularities? Why isn't this theory taken more seriously? So this is an interesting question.

0:56:56.8 SC: So let me get a little more technical than average here. Sorry for getting technical a couple times this early in the AMA. But, you know, it's good. You don't need to pay attention if you don't want to. So when you think about general relativity, what did Einstein do? Einstein says the curvature of spacetime gives rise to gravity. How do we describe the curvature of spacetime? We have a field called the metric tensor field, which is a field just like the electromagnetic field. It has a Lagrangian. You can vary it. You can get equations of motion. You solve those equations of motion. You get the gravitational field. That's what people like Einstein did and then Schwarzschild did for the Schwarzschild solution and so forth. And it turns out this was a perfectly sensible thing to do based on what Einstein learned about Riemannian geometry from Marcel Grossman, et cetera. But geometers are going to keep thinking and they have kept thinking. And what you learn when you take your general relativity course is that there's actually a two-step process to deriving curvature from the metric.

0:58:00.0 SC: The first step is that you take the metric and you derive what is called a connection field or the Christoffel symbols or the affine connection, different words attached to the same basic idea. It's this connection field that allows you to parallel transport, as we previously mentioned, vectors from point to point. And then from the connection, you derive the curvature tensor, okay? And this is provocative because there are other theories that we know and love, gauge theories, right? Electromagnetism, QCD, the weak force, the standard model, et cetera. These are all gauge theories. They do not have a metric in them because they're not theories of space time. They're theories of internal spaces like color space or the phase that you have in electromagnetism.

0:58:48.6 SC: So in those cases, you start directly with the connection and then you do the same kind of mathematical manipulations to get the curvature that you do in general relativity. You just don't derive the connection from a metric. You posit it directly as your fundamental field. All that's great and that's fine. Nothing incompatible there, just an interesting, amusing observation. So Elie Cartan, who was a brilliant mathematician, thought about this very carefully and realized that you could break the connection between the metric and the connection as it were. I didn't mean to construe that in that way. I didn't mean break the connection. I mean break the relationship between the metric and the connection.

0:59:32.2 SC: You can, in other words, treat the metric and the connection as both independent variables. You're allowed to do that. Just posit there's a connection field and there's a metric field and then derive some equations of motion for them, okay? That's a different theory of gravity, the Einstein-Cartan theory of gravity. And there's different ways to do it, actually. You know, the Einstein-Cartan theory is one way to do it. There's other ways of doing this thing. The whole thing is now called the Palatini formalism for deriving equations of motion in theories of gravity. And one thing that can show up and often does in this approach is what is called the torsion tensor.

1:00:13.0 SC: Torsion is a part of the connection that is not related to the metric. So it's an independent thing. So there's the part of the connection that can be related to the metric and a separate part that is not, which is the torsion tensor, okay? So in some sense, Einstein-Cartan theory is one where torsion is a new ingredient in the theory. One is tempted to say a new degree of freedom. But what Cartan did, Cartan not being a physicist, I don't actually know a lot of the history here. All I know is, you know, what happens, this is 100 years later after all this was going on. So I'm reconstructing a little bit. But Cartan took the same starting point for the action, the thing you're going to vary to get the equations of motion as Einstein and Hilbert did. And he varied that. And so he got certain things. And in his way of doing things, the torsion exists, but it is not an independently dynamically propagating degree of freedom.

1:01:10.1 SC: You can actually relate it directly to what other fields are doing. So in fact, the torsion is going to depend in Einstein-Cartan theory on the amount of spin that you have in other particles in the universe. So if you're in empty space in Einstein-Cartan theory, the torsion is always zero because there's no particles, there's no spinning there. If you're inside matter, then the torsion is not going to be zero. So this is a choice to make the torsion a non-propagating field. You can also write down theories where the torsion is a propagating field.

1:01:44.8 SC: Neither one of these two ideas is a very good idea, or at least not a very good idea as a physicist, or at least maybe it's a good idea. But it's not one that really pans out in any obvious way. So let me try to explain why. If you make Cartan's choice, which is the torsion is not propagating, it's sort of a variable, but it's literally algebraically related to the other one. So there's no independent dynamics there. Then you can always do mathematical manipulations to eliminate it. You can just get rid of it, right? If I say x, let's say x is the torsion and there's some other variable from the matter field called y, and I say x equals y, right? That's the equation of motion. It's not derivatives of y or anything like that or derivatives of x, it's just x equals y. Then everywhere I see x later on, I can just replace it with y, right? I don't need x. It's just equal to y.

1:02:36.8 SC: That's what is the situation in Einstein-Cartan theory, is because it's non-propagating, you can just solve for it and get on with your life. So what a physicist would say, I would argue, is that Einstein-Cartan theory isn't a separate theory of gravity. It's just ordinary general relativity with some specific choices about the energy-momentum tensor or about the behavior of matter, if you want to put it that way. And therefore, the idea that this theory avoids singularities seems bizarrely off-base to me. I don't quite believe it. I'd not heard that claim before. After you asked this question, I googled it and there's a Wikipedia page on Einstein-Cartan theory and it does mention this claim that it avoids singularities. And if you dig a little bit deeper into this claim, they all reference papers by one person.

1:03:30.7 SC: So there's one person out there who thinks that Einstein-Cartan theory avoids singularities. They might be right, but it goes against everything that I know about how gravity works. So I'm going to bet that it's just not correct and that's probably why it's not taken more seriously. But I want to mention, I'm glad you brought it up, because there is the other option, right, which is to make torsion dynamical, to let it be, you know, a propagating degree of freedom. And then it's interesting 'cause lots of people have pursued that also. You know, this is not a new thing. And I got very interested in this in graduate school because I was interested in general relativity. I was, Ted Pine, my graduate school collaborator and the musician behind the Mindscape intro and outro music, he and I taught to our fellow graduate students a little seminar in general relativity and we were thinking about it along the way and thinking about torsion 'cause we were very interested in geometry and so forth. So, but I knew a little bit of quantum field theory too.

1:04:32.0 SC: And a lot of the people who get interested in torsion are not quantum field theorists at heart. They're either geometers or general relativists. So here is how a quantum field theorist thinks about this. I don't care where your idea for a field comes from. What I care is how the rules of effective quantum field theories treat your field. So you tell me you have a torsion field. You tell me you're inspired by letting geometry play a role in your theory. Great. Good for you. But at the end of the day, I have a Lagrangian that defines my field. I have an effective field theory for it. I'm going to treat it like any other field. There are rules to effective field theory.

1:05:13.2 SC: Once I've written down the fields, the degrees of freedom and the symmetries that govern their interactions, I don't have any more choices that I'm allowed to make. Every possible interaction should be there with certain coefficients and, you know, some coefficient values might be natural or unnatural but we have expectations for what they should be. So I became very interested in this and actually I will tell you a personal story that I haven't told many times just 'cause it doesn't come up, not 'cause it's especially embarrassing. It's only a tiny bit embarrassing. I was a grad student while I was working on this and I was on the postdoc job market and I gave a talk at the Institute for Advanced Study where I was a postdoc candidate and I talked to Edward Witten about what he was working on and what I was working on.

1:05:57.5 SC: They were very different things, of course, and I mentioned that I was interested in torsion theories and he said, I can tell you something interesting about torsion theories, you know, slightly with some trepidation. I said, what is that? And he said, they don't exist. And actually to go back to the previous question about being intimidated, here's an example where I was intimidated. I was much younger and more foolish then, right? Even younger and more foolish than I am now. And so I didn't press him on this. He just gives this sort of Delphic utterance that these theories that I've been interested in don't exist. I didn't know really what he meant, but I figured probably I should have known what he meant.

1:06:34.7 SC: I was on the job market. I didn't want to admit it. So I didn't press him on what exactly he meant. I think now I know what he means and I think he's not right because here's what I think he means. I know I'm getting into the weeds here, but you can indulge me a little bit. I can barely talk, so you can at least indulge me when I want to talk about something. When you have a tensor, like the Riemann tensor we mentioned earlier has four indices, the torsion tensor has three indices. What that means is there's a lot of different components. It has three indices in spacetime. Spacetime indices take values from zero to three, four different possibilities. That's four times four times four components in the tensor, okay?

1:07:16.9 SC: And that's a lot. And it's actually less than that because there's some symmetries that relate them, but still there's a lot of different components. When you try to write down a physically sensible dynamical theory of such a tensor, what's going to happen is you will be constrained by requirements like Lorentz invariance. This goes back to what we talked about in Claudia Durham's podcast, the huge number of constraints and specific requirements you have to satisfy when you're inventing a new theory of gravity. And what happens generically, if you try to write down torsion and let it be dynamical, if you, the energy, the kinetic energy associated with some of those degrees of freedom in the torsion tensor will have the opposite sign of the kinetic energy associated with other degrees of freedom, which means that if you didn't try too hard, you would have some positive energy degrees of freedom and some negative energy ones, and negative energy degrees of freedom are just bad.

1:08:19.2 SC: Sometimes this is phrased as having a negative norm on Hilbert space, but I think that's not the right way to think about it. I think that, and this is a paper I wrote with Mark Trodden and Mark Hoffman about phantom energy in the context of quintessence and dark energy a while back, but you should actually just think about these wrong sign modes as physically possible, but having negative energies. And what that means is empty space becomes wildly unstable, right? You can just explode the vacuum into a bunch of positive energy particles and a bunch of negative energy particles. So it's not mathematically illegitimate, it's physically ruled out. I think that's what Edward was getting at with the torsion theories don't exist.

1:09:01.6 SC: But if you think about it a little bit more, you just need to be a tiny bit clever and what you can do is just set some of those degrees of freedom to zero, okay? So even though they might want to have negative energies in their kinetic energies, as long as they don't propagate at all, that's not a problem. You can pick and choose Lagrangians for these fields where some of them propagate and some of them don't, okay? And so I wrote a paper about this with George Field, Consequences of Propagating Torsion in Connection Dynamic Theories of Gravity, something like that. So theories of gravity where the connection is independent of the metric and we have this kind of deflationary result which is sure, you can write down Lagrangians, you can have dynamics for these fields, bless your heart, you can do whatever you want.

1:09:48.4 SC: But from the point of view of effective field theory, there's no reason, there's no symmetry that prevents these degrees of freedom from having a mass. It's not like fermions which have chiral symmetry or gauge bosons which have gauge invariance or whatever. They're just scalar fields or vector fields or whatever with no symmetries protecting them from getting a mass and in effective field theory, the mass should be big. It should be the Planck scale or the GUT scale or some large number. And if that's true, these particles just decay away in the very early universe and they have absolutely no effect on our universe today, on its dynamics, anything like that. They're like other GUT scale or Planck scale particles. Maybe they exist but there's no way of us knowing.

1:10:30.4 SC: So my view on torsion theories is that either the torsion is non-propagating and then it's really just general relativity in a particularly unelegant way of writing it or torsion is propagating but it's completely invisible because it's all propagated and decayed away by now. I don't think there's anything more to be said about them. Sorry about that. I guess that is kind of a catch phase now that I think about it. Sorry about that. Okay. Simon Carter says, will you be coming to the UK this year as part of your Biggest Ideas in the Universe part two book tour and have you chosen a topic to present yet? I will not physically be visiting the UK. I'm trying too hard to write volume three. So I'm trying not to do too much traveling but I will do a virtual talk via the Royal Institution I think. I don't know what the date is. Maybe I can look up the date. Let me see if I can look up the date here in real time. Probably not. Probably not. Yeah. Let's see. When will the date be? I don't see when the date will be. It's probably not going to be until June or something like that. So I can't... I just don't know.

1:11:33.5 SC: But look out. I will try to. I do have a calendar on my website. I rarely update it but I will try to do that. But there will be a virtual talk. Topics to present, yeah, quantum field theory is very, very rich and so I need to pick something specific to talk about. I need to pick something that is cool sounding enough that people will come to the talk but different and fun enough that they won't just hear a talk they've heard a million times before. My tentative guess is that it will be on effective field theory, right? How field theories can be so good even though we don't know what happens at infinitely small length scales, et cetera. Why we can ignore the torsion tensor. There you go. That actually comes right out of effective field theory. That kind of thing is probably what I would like to talk about.

1:12:23.4 SC: Callan says, priority question, what would you say to someone who is struggling with anxiety or angst over the implications of naturalism? I'm not so scared of dying, oddly enough, but I do find it difficult to accept determinism. For some reason it just bothers me. I suppose it undermines my sense of control over my life. I really look up to you as someone who's helped guide me in life through your work and you seem to be an optimistic, easygoing guy so I'm just wondering, did you ever struggle with angst like I'm feeling much?

1:12:54.0 SC: So I get it. I mean I'm not going to disparage your feelings of anxiety. Those are very real. Naturalism and the implications of naturalism have never personally bothered me in that particular way. So I might not be the best person to talk to actually in terms of thinking through those things. Just so we're super clear because we just did talk about determinism a moment ago, our universe is not deterministic for all intents and purposes, right? Quantum mechanics gets in the way. But that's not, it's also not what is relevant to this question. What is relevant is are you obeying the laws of physics? Whether the laws of physics are themselves deterministic or stochastic in some sense is not the point as to whether or not there is some libertarian kind of free will that lets you control your life in a way that is not settled by the laws of physics and naturalism is certainly going to say that's true. I guess it shouldn't undermine your sense of control over your life.

1:13:56.3 SC: I think about it this way. Here is... You don't know this but right now as I'm talking, I'm looking at my coffee mug, Okay? And I can imagine picking up the coffee mug and moving it an inch to the left or moving an inch to the right. Indeed, I just did. You can hear it banging on the table. It's a metal coffee mug in case you're wondering. So there's... If I wanted to say, no, I would rather have moved it to the right first than to the left, there's nothing stopping me from doing that, right? And you can say, well, but the laws of physics will say that you're going to, who cares about what the laws of physics are going to say? I don't know that. Nobody else knows that. That is entirely irrelevant. If I sit here and look at the coffee mug and say, I'm going to move it one inch to the right and I sit here and contemplate that I'm going to do that, nothing is going to stop me. You can't stop me. The world can't stop me. Laplace's demon can't stop me. Watch. There. I just did it, right?

1:14:57.5 SC: That's why I need to emphasize like I just did above, this higher level emergent blah, blah, blah. This is not just copium. This is not just saying, well, for all intents and purposes, blah, blah, blah. This is the real world. This is the world in which we live. The fake world, it's not fake in the sense that it's not real. But the world that we have no legitimate access to is the Laplace's demon world where we imagine that we're super powerful godlike beings and know what every single atom in the universe is going to do.

1:15:29.7 SC: That is mathematically a very nice way of describing the underlying workings of the universe and indeed, I have devoted my life to studying it. But it is irrelevant to getting through your everyday life. That certainly it shouldn't make you sad that there exists such a hypothetical determination of what you're going to do next. I mean I've used this analogy before. So stop me if you've heard it. But I think a lot of people hear about determinism or the laws of physics and they kind of think of it like Harry Potter or Shakespeare's witches who are giving prophecies about the future and they say like, you're going to do this.

1:16:13.8 SC: This is going to happen and there's nothing you can do to prevent it, right? That is absolutely not what the laws of physics are like because the laws of physics might very well say what is going to happen next but not in a way that you can ever know. So the best analogy for the laws of physics is not like super intelligent wizards telling you what the future is going to hold but an annoying little kid and the kid says, I know what you're going to do next and you say, Okay, tell me what I'm going to do next and the kid says, oh, I can't tell you and then you do something and the kid says, yep, that's what I knew you were going to do. Would having such a kid tagging around really change your outlook on life in any way? I mean it would be mildly annoying but it wouldn't make you do things or not do things or feel you didn't control your life in any way. To the... Let's put it this way.

1:17:05.9 SC: If you think it's legitimate to use words like myself, my thoughts, my actions in the world, all of those words are embedded in the higher level theory, right? None of those words are there in the standard model of particle physics. So as soon as you use that language, the rest of the theory comes along with it including words like my choices, the consequences of my actions, the effect I have on the world. All those are truly, really, honestly, not just telling you this to make you feel better, part of the world. That is the way I personally look at it. Danielle Lantz says, can you explain the preferred basis problem? Do you think it poses an actual challenge to the many-worlds interpretation? I mean as you might guess, no, I do not think it poses an actual challenge.

1:17:58.0 SC: Or at least, let's put it this way, there are challenges and there are puzzles. Puzzles are just resolved, right? Resolvable, I should say, not resolved. Puzzles are good. Puzzles make you think more deeply about the theory and go, oh, I wonder how this fits in. So the preferred basis problem has been solved already. And to be honest for those of you who don't know think about Schrödinger's cat. So I'm gonna assume the basic setup I will even go so far as to assume you know that I like to put the cat in a super position of awake and asleep rather than alive or dead. But the point of Schrödinger's cat is to take a microscopic quantum superposition and amplify it to a macroscopic quantum superposition. So we're very familiar with microscopic particles and atoms being in super positions of different possible observables.

1:18:43.6 SC: But in the macroscopic world, we don't see cats in superpositions of awake and asleep. We only see them either awake or asleep. So in the Copenhagen interpretation, you say, when you measure the cat, its wave function collapses onto either awake or asleep with some probability. In many worlds, you say it branches. And the preferred basis problem is why those branches? Why does it branch into specifically, I see the cat awake, or I see the cat asleep. Why isn't there any branch where I do see a macroscopic superposition? And I think this has been completely a 100% solved by the idea of decoherence and pointer states. It took work, right? This is not exactly trivial. You have to actually think it through People like Heinz-Dieter Zeh and Wojciech Zurek, and Jim Hartle and Murray Gell-Mann, et cetera. Thought about these problems very hard and it solved the problem.

1:19:47.8 SC: And the answer is that there is an environment in the world. The environment is everything else in the quantum state that you're not paying attention to. And the environment is the photons in the room, the atoms of air, et cetera. Not to mention the experimenter, the apparatus that you're not including in your quantum description, et cetera. Literally everything else. And decoherence happens because the environment interacts with the system and becomes entangled with it. And the pointer basis problem is solved. Because if you start with a cat in a superposition of two different configurations, which are macroscopically distinct, right? So one is the cat's over here, the other's, the cat's over there, that will become entangled with the environment right away because photons will interact differently with the cat, whether it's either over here or over there. A photon that is absorbed by the cat if it's walking around, will miss and not be absorbed by the cat if it's lying on the ground asleep.

1:20:48.3 SC: Okay So that's a specific kind of entanglement that occurs, and that's what branches the universe into those two possibilities. Because once you're in one of those possibilities, once you're in the branch where the cat's just awake, there's no more entanglement that you get. The photons keep hitting the cat, but every photon hits the cat in the same way. 'cause it has one definite macroscopic configuration in space. Therefore, you're in a branch and you're staying in that branch. That's why we have a preferred basis where things look spatially localized and coherent, because those are the structures where entanglement doesn't keep decohering the universe. Scott Stombaugh says priority question. There's a philosophical position you may be familiar with called Antinatalism. The idea is that human procreation is much more ethically tricky than people appreciate. Antinatalists like David Benatar argue that every human life experiences mild to severe amounts of suffering, and that no human life consents to their own existence.

1:21:45.5 SC: Looking at the history of humanity so full of profound suffering for certain individuals and groups, I have to admit that it's hard for me not to be sympathetic to these arguments. I'm wondering what your thoughts are on antinatalism and what points you might make in debate with someone who holds this position. I think the major, so I'm not a fan of antinatalism, and I think that the problem with it comes right up in your two arguments for it. Like you presented two arguments. One is every human life experiences mild to severe amounts of suffering. And the other is no human life consents to their own existence. So these arguments are incompatible with each other. If you think about it, because the first one, every human life experiences mild to severe amounts of suffering. That's by itself not an argument.

1:22:35.4 SC: There's the hidden implication of it. And that is bad, right? And in fact, presumably that every human life, I mean, I guess you're trying to say that there's more suffering than happiness, right? Which is a judgment call. Maybe it's true for some lives, maybe not for others. So I don't want someone else deciding whether my life has more happiness in it or not. That should be my choice, right? And you say no human life consents to their own existence. That's certainly true because when they were brought into existence, they didn't exist. When that decision was made, how could they possibly consent? That makes no sense. In order for them to have the ability to consent, they need to not only exist, but have certain cognitive capacities, et cetera, which don't come into being until long after they technically began to exist.

1:23:23.9 SC: So if you care about human beings having the right to consent to things, you have to have them exist first. And once they exist, they should be the judge of whether or not there is more suffering in their lives than not. Very few antinatalists actually end their own lives, right? They just argue they should not have been brought into existence in the first place. If you want to make an argument that it should be easier for people in sound, mind and body to choose to end their lives, I would be sympathetic to that. But that's at least a coherent argument. The antinatalists one, I think is sort of self-contradictory. Bob Brichi says, regarding the Fermi paradox, I keep reading the self-replicating Von Neumann probes would be easy to produce. Is that really true? Mining asteroids and creating free new probes with sophisticated propulsion, navigation and communication systems saturating the galaxy and waiting thousands of years for reports.

1:24:21.1 SC: Your thoughts? Yeah, I'm on the side that it'll be easy to produce right. Now easy, of course, is highly relative because it would be very hard for us to do it with our current level of technology. It's a major achievement just for us to get something to the moon. We haven't gotten a human being there in quite a while, much less the other side of the galaxy. But we are nothing we're nobody. I mean, when you are thinking about the technological capabilities of alien civilizations, the best... Let's put it this way, the fairest assumption to make is that such civilizations would be at least a billion years more advanced than us. It could be several billion years, but I'm trying to be conservative here. An average, if you are believing in a scenario where there's lots of civilizations that come up, it's not very rare then the galaxies, many billions of years old, they should have come up a long time ago.

1:25:19.0 SC: And so there are a billion years more advanced than us, so they have more technological difference between them and us than between us and earthworms, right? So since we can even see the vague outline of how to do this for a civilization with a billion years more practice and time to learn, it should be completely trivial. Yeah, I mean, as long as it's not against the laws of physics, I think that something like that should be very, very doable to advance civilization. Charles E. Grant says, I'm fascinated by the idea that space time emerges from entanglement, but I'm confused In the laboratory, we can maximally entangle two particles in a bell state, and then we can separate those two particles by as great interval as we want. We could also do this with two particles that are completely unentangled. Naively, this would seem to indicate that entanglement and space time separation are independent.

1:26:11.8 SC: Yes, all this is very true. So there's two responses to it. One is when you... Well, there's three responses to it. First, to be honest to what the rest of the field is thinking about. Most physicists, when you talk about the idea that space time is emerging from entanglement the entanglement and the space time exist in two different worlds, two different spaces. Not many worlds of quantum mechanics worlds, but they're thinking of AdS/CFT. So they're thinking of a setup where you have a negative cosmotial constant in a space time, and then it has a boundary with one dimension less where there's no gravity. And the entanglement is in the boundary theory. The theory without gravity and the space time is in the bulk theory. So you can't actually think about the entanglement between two different points in space giving rise to the metric in that space, in this way of thinking about it, they exist in two different mathematical descriptions.

1:27:09.9 SC: Now, what I and my collaborators, Charles Sao most obviously have been thinking about is, can you do it as we say in the Bulk, Bulk entanglement, gravity? And the reason why this is something that is reasonable to consider is because we are not trying to think about cases where the gravitational field is large. We're not trying to think about horizons or cosmology or black holes or anything like that. We're just trying to think about the regime where gravity is an ordinary local field theory like it is here in the solar system. For example, why do apples fall from trees? And so in that case, we propose that it's perfectly plausible to do local entanglement. And then indeed then you're closer to what you're worried about. And then I literally am saying that in two different regions of space, the space in which we live, there's an amount of entanglement, which is closely related to the metric on space time, and from which you can derive one from the other.

1:28:06.7 SC: But the clarification here is that the entanglement we're talking about is not the entanglement between particles. We're talking about entanglement between the vacuum. We're talking about entanglement between empty space regions. So I take two regions of space. There may or may not be particles in them, but here is a mathematical fact about quantum field theory. In any region of space, you can think about many different modes of the quantum field, okay? We say that there are modes in the vacuum if there's nothing going on there, if there is a particle, then some particular mode is excited. A mode is just a part of the quantum field with a fixed wavelength of vibration. So one wavelength is excited, or some combination of wavelengths are excited, but other modes are still in their vacuum even in super dense matter, even in like the center of the earth or whatever.

1:28:58.9 SC: The vast, vast majority of all the modes we can think about in the quantum field theory are still in their vacuum state. That's just a mathematical fact about how things work. If you tried to excite too many modes, you would collapse to a black hole. So it's just a feature of quantum field theory that no matter whether there's matter there or not, most of the quantum field modes are in their vacuum state. It's the entanglement between the vacuum state modes that we're talking about when we say the metric on space time arises from entanglement. So sure, you can entangle two particles, but that is completely irrelevant. That is completely swamped by the entanglement between the vacuum modes. So that was the second thing to say. So that's why it's not in any sense, a contradiction. The third thing to say is more ambitious thinkers than us, such as Juan Maldacena and Leonard Susskind have proposed that in some sense, there is a little space time wormhole connecting any two entangled particles.

1:29:56.7 SC: That is the ER equals EPR conjecture. The place where that conjecture is well understood is far, far away from just two particles in a bell state. But a very, very large number of degrees of freedom entangled with each other creates a macroscopic wormhole. These microscopic wormholes between just two entangled particles are a bit more conjectural and harder to understand, but maybe they're there, or maybe they're there, but in a different sense than we think. I'm just really not sure what to think about that. The Great Deceiver says, when is a good time to give up on a dream? Have you ever had to give up on a dream? Here in Canada among several other crises, there is a housing crisis even more acute than in the US being a bit late to the home ownership game, I now find myself being forced to give up the dream of ever owning a little house as a single middle class earner, it is simply not possible.

1:30:48.8 SC: The average home here costs about 700K. There are millions in the same position. It's just kind of sad. But here we are. This is... I mean, I think as you probably can guess, this is not something about which general purpose advice is give-able. I mean, I get it. I feel bad. I think that you're... Like you say, you are in the same position as millions of people. Owning a home is a very nice thing. I'm fortunate enough to be able to do it. But many people are not. And I am not happy that it is so hard. I wish the system we were in was different so that it would be easier, most obviously just by building a lot more housing so it'll become more affordable. But your question is not specifically about finances so much as about psychologically when to give up on a dream.

1:31:37.2 SC: And the problem is you're talking about probabilities, right? About propensities and chances and unknowns, not about definite things. If you knew that something was unachievable, then you should give up on it. That makes sense. But you don't know, like maybe housing prices come down, maybe your income goes up. Maybe you could just really, really try to save for a long time and make it plausible. It's just impossible for me to say what the actual sensible, pragmatic thing to do here is. But let's imagine that you've convinced yourself that it's just not gonna happen, right? I don't know if that's true. I'm not saying that that's true, but let's imagine that were true, that you've convinced yourself that it's not going to happen. Then I think the trick is to shift dreams, right? I mean, it's nice to own a little house, that's great, and that's a good dream to have.

1:32:29.8 SC: But there's other dreams to have. There's other ways to have a fulfilling successful life within the constraints of what you're able to do, right? I mean, the system might not be fair. The system might not be set up to reward equal effort with equal rewards. You might be doing as as much as you can and still fall short of a particular dream. But there's the parts we can control and the parts we can't control. And so, by all means, try to fix the system, but meanwhile, try to adjust your dreams to things that have plausible chances of occurring, you know they're dreams. So almost by construction, they're at the upper level of things that have plausible chances of occurring. But I do think it is possible to change our perspective on what's important and what's not in conditions where something that we thought was really, really important just isn't going to happen.

1:33:26.5 SC: I mean, your dream is one that I really do think that in the world it should be much more plausible for almost everyone to achieve. Some people's dream is to be a famous rock guitarist or baseball player or whatever. And that's a situation where many, many dreams are not fulfilled, or even for that matter, become a tenured, theoretical physicist, right? The number of people who want to do that is much larger than the number of people who eventually will. So yeah, I mean, giving up on dreams is something that many, many people are going to have to do. I'm kind of annoyed that in the public discourse we put so much emphasis on just trying with all of our might to achieve our dreams, and maybe that will make it happen. There's a huge selection bias from the people who are saying this kind of thing that either they're hucksters who just trying to make a buck by saying that, or they're people who have achieved their dreams and therefore they think it's possible 'cause they did it, right?

1:34:27.9 SC: I think the number of big amazing dreams is much larger than the number of achievable dreams. And that's okay, right? I mean our reach should exceed our grasp a little bit. We should always aim a little bit higher than we can get. And when we don't get it, we need to find contentment and fulfillment doing a little bit less than that. Again, I think a house is something that is not in any sense an unreasonable dream, but given that we can't necessarily control that, you have to control, what you can control, and that is your personal sense of fulfillment and contentment, it is absolutely possible to be happy without having a house. I mean, I was in that situation for much of my life, right? And there's other things that can make you happy.

1:35:15.6 SC: So sorry to hear about the situation you're in, but I do hope that you can sort of shift into a mode where you're finding fulfillment and contentment in other ways. That's the best I can say. Sorry. I'm gonna group two questions together. One is from Abanish Nala who says, do you think that spin glasses are appropriate models for real life complex systems such as societies and economies? The other is Herbert Berkowitz says, in your recent solo episode, you mentioned Econophysics, it is an apparent attempt by physicists to apply the principles of physics to the field of economics. Do physicists really think that they have something to bring to that table? What does the behavior of particles have to do with behavior of people? So yes, Herbert, I do think that physicists do think they have something to bring to that table, and I think that they do have something to bring to that table.

1:36:06.5 SC: What does the behavior of particles have to do with the behavior of people? Well, they're both physical systems that obey the laws of physics. In particular, there are things that physicists learn that are more or less independent of the details, right? Especially in statistical physics. Ideas like entropy and information and phase transitions can absolutely be borrowed from physics and applied to other circumstances. It's not a matter of arrogance, it's not a matter of physicists think they're so smart and they can fix everything. It would be completely false to think that the best way to study economies or societies or whatever, is to think like a physicist. There are specific problems in those domains that you have to think like a domain specific expert to have any handle on. That's not the idea. But the idea is that physics is easy. Physicists study simple systems.

1:37:04.5 SC: Atoms are much simpler than people. And there is a special kind of insight that you learn from studying the simple systems, from studying the spherical cows, where you've abstracted away all of the complexities because the systems are so simple. If you can't understand them, then you have no hope of understanding the more complex ones. Even very, very simple systems, when you have many, many moving parts in them, become very rich in their behavior. And so you can learn things by studying those simple systems, and then you can extrapolate those ideas to more complicated, realistic, complex, messy real world situations. You have to do so cautiously because the introduction of non sphericity into your cows might be crucially important for all sorts of reasons. But you can ask the people who do these things have these ideas had good impact. Thomas Schelling won the Nobel Prize in economics for ideas that were very closely influenced by physics and things like the Ising model and stuff like that.

1:38:08.8 SC: So I think that empirically it does work and we'll have to see how it goes. Abanish is asking specifically about spin glasses. Spin glasses are an interesting case. They were a recent Nobel Prize for Giorgio Parisi who studied spin glasses. And actually he has a book that is out about his adventures studying spin glasses and other physical models for complex systems. So a spin glass. What is a spin glass? If you know a little bit about the kinds of toy models that physicists study, the Ising model is a very famous one. The Ising model is one where you have a lattice and there's some spins at each vertex of the lattice, and they have an interaction between them. Either they're attracting each other or not. I shouldn't say attracting, they're trying to line up. So the spins want to be in the same direction, or they're trying to anti line up, so they want to be in the opposite direction, okay?

1:39:06.3 SC: And that makes... That's a perfectly good system to study. You can also put it in a magnetic field, you can put it in a temperature bath. Lots of fun things you can do with it. A spin glass is kind of like that, but instead of having a uniform connection between the different spins where they all wanna line up, or they all want it to be anti-aligned, you have random connections between one spin and the next. So between any two spins, there's some interactions, some of them are the spins wanna line up. Some of the interactions are the spins wanna be opposite, okay? So if I take one spin and it's surrounded by, let's say four others, and for three of them they want to line up, and from one of them, they wanna be the opposite. You might wanna say, okay, I am gonna start, I'm gonna take my first spin, set it up, pointing upward the ones that want to be aligned with it.

1:39:58.9 SC: I will also put upward and the ones anti-aligned, I'll put downward. But then you go off to one of the ones that wants to be up and you say, well, it has four neighbors, and what about the other three neighbors? They want to do something different. And what you'll quickly find is that there is not in general any way to satisfy all the desires of all the spins all at once. There's going to be some frustration as it's literally called frustration. And so what this means is it's not at all obvious what is the lowest energy state of this system. And in fact, it's going to be true generically. There are many, many, many states that are relatively low energy. In other words, there's many states that are not the lowest possible energy, but that flipping any one spin increases their energy rather than decreases them to truly decrease the energy.

1:40:48.8 SC: You would have to flip a whole bunch of spins in concert in some complicated non-local way. So these systems exist physically, and you can study them spin glasses, and they can sort of gradually quantum tunnel down to lower and lower states, but they might never reach the bottom. That tunneling time might be very, very, very, very long. So you get all sorts of sort of complex dynamics in what happens to these spin glasses. So the suggestion has been made that this kind of complex dynamics might serve as a model for real life systems of people or financial institutions or something like that. So I actually don't know of any specific examples where I would say this is a great model. That is not to say they don't exist. I have just not thought about it very much. I think that the better question... Well, let's put it this way.

1:41:42.8 SC: It's too easy to have a system that you've studied a lot and say, well, maybe this is a model for everything, right? The trickier thing, but probably more valuable is to take the system that you know very well and say, okay, which aspects of this system can I port over to the more complicated real world complex system that I care about? Maybe the answer is all of them, none of them, some of them who knows. I mean, there's absolutely an extra step that needs to be taken very, very seriously when you try to be someone inspired by physics, but saying interesting things about complex systems like societies and economies. So in the case of spin glasses, I'm not sure, but it's plausible. I would be very interested in any specific examples that anyone knew about Sean And Kana says, given the surprising findings in quantum biology over the past few years, do you think there may be silent quantum effects in the brain?

1:42:38.8 SC: Are quantum effects likely to play an essential role in perception, cognition, or consciousness? Who knows? I really don't know myself. It's possible one's, first guess is no, that quantum effects are not important here. And the reason why is decoherence, which we just talked about, the brain is a warm, wet system as people like to say. It is a system in which different parts are constantly jostling into everything else. Think of a quantum computer. If you wanna build a quantum computer, you need a gigantic apparatus to just get a tiny number of qubits. And the reason why is because you want to shield those qubits from decoherence. So you need to cool them down to an incredibly low temperature. And well below zero, let's put it that way, a few degrees above absolute zero maybe, and then you need to keep them close enough to interact, but far enough away so they don't decohere each other, et cetera.

1:43:35.5 SC: None of these things are going on in the brain, right? Our brain is not trying so hard to shield itself from decoherence like effects. And if you don't do that, then any quantumness this is not going to play a primary role. So your guess would be no. On the other hand, you might say the same thing about photosynthesis and other examples where quantum effects do seem to be important. So I'm open to the possibility. I think that one's the betting money should be on quantum effects and not being very important in the brain, but biology is complicated, physics is interesting. We'll have to do the experiments and take the data and we'll have to see. Frank Schultz says in episode 110 with Neil Johnson on complexity, conflict, and Infodemiology, you were discussing the possibility and feasibility of controlling eg. Radical fringe groups and the spread of misinformation in information networks in the modern world, social media, et cetera.

1:44:33.8 SC: I was a bit shocked during that conversation because neither of you were realizing how being able to control fringe ideas and boiling up the social dynamics in these networks can be abused by eg. Totalitarian regime. Shouldn't we be more careful when doing this kind of research? What happens if it falls into the wrong hands? Well, I'm not exactly sure what is meant by the question at the end there. You know it's absolutely true that both the existence of social media and different ways of trying to stamp out misinformation in social media can be abused by totalitarian regimes. That would be... That's absolutely something that is true. Like many other things fire and the steam engine can be abused by totalitarian regimes also. But I'm not sure what you mean by careful to worry about what happens when it falls into the wrong hands.

1:45:28.1 SC: Are you suggesting that we shouldn't do the research or that we should do the research, but then keep it secret? And if you do... If you are suggesting that, then I would begin to worry who exactly is the totalitarian regime here? I tend to think that this kind of research into complexity and Infodemiology is academic research that should be shared as widely as possible. I think that in general aside from some very specific examples of truly weapons oriented research, that human knowledge should be spread widely. And of course, it can very often be misused, but that is something we should learn to deal with, not by suppressing the knowledge, but by trying to decrease the number of totalitarian regimes out there. Or trying to pressure them into not misusing knowledge or something like that, rather than suppressing the knowledge itself.

1:46:25.0 SC: That would be my particular way of going. Nathan says, how has the Francis Scott Key bridge collapse affected you or those or... Yeah, you or those, you know well, I hope all is well. So yeah, nothing specific for me. Here in Baltimore, we are very concerned that we had this accident that destroyed the middle section of the Francis Scott Key bridge. The bridge for those of you who don't know, we have a harbor that goes all through the south part of Baltimore, like it goes into Baltimore. The apartment where I lived when I first moved here was looking over the harbor. The inner harbor is the great place to come visit if you're ever visiting Baltimore. It's not actually near where I live now which is near the Hopkins campus. So I'm not directly affected in any way.

1:47:14.8 SC: I think that if you are a commuter, or much more importantly, if you are a truck driver or someone who does transport on highways, then that bridge was crucially important to get from north of Baltimore to south of Baltimore. And even more importantly the collapse of the bridge has really closed down the Port of Baltimore for all intents and purposes for a while. And that's a lot of jobs and also just a lot of activity one way or the other, whether it's cruise ships or cargo ships or whatever. So Baltimore has been hit by this, like someone estimated $2 million a day is the economic loss to the city. I don't know whether that's the entirety of it or just actual dock workers, I really don't know. But the point is, it's bad.

1:48:08.3 SC: It was very impressive to see right after the event happened to people sort of reconstructed the chain of events. And it seemed, as far as we can tell, it's not any particular person's fault. What happened, the ship lost power. And therefore as someone who's done a tiny bit of boating in my time, if you've not ever piloted a boat and all you've ever done is driven cars or bikes or whatever, being on water is very different in feeling than being on land because you can't stop. I mean, you can stop, but then if you don't control it, you are getting pushed around by the motion of the currents or by your inertia or whatever, right? It's just a very different sense of physics to be on the water than to be on the land.

1:48:56.8 SC: And it was a matter of 30second intervals between the power being lost on the ship, the captain declaring a state of emergency them contacting the bridge officials, the bridge officials seeing the problem, and then shutting down the bridge. Everything was amazingly efficient. Nobody at any chain, any step on the chain said, well, I don't know, this might be a public relations disaster or anything like that. I mean, kudos to everyone in that particular set of events. At the end of the day, the ship still hit the pylon and destroyed the bridge. But still they were doing everything that they could in the middle of the night to prevent anything worse from happening. Of course, I don't know why power went out on the ship, so that might be someone's fault. I truly don't know.

1:49:45.8 SC: So Baltimore's a big city, or at least a medium big city and lots of things going on, and many of them had nothing to do with the harbor or that particular road crossing the bridge. So if it weren't for the internet, I wouldn't know that the bridge had collapsed. So no, it does not affect me or those I know well. It does affect people though. And Baltimore is for various reasons an awesome city in some ways, a struggling city in other ways. We don't need more struggles, to be honest. So any little thing like this is like ah, geez, do we really need this right now? Just like the pandemic hit the Baltimore restaurant scene really hard. When Jennifer and I moved here and we were looking at restaurants we would... On the internet, we were like oh, this one looks good.

1:50:35.1 SC: Oh, no, closed because of the pandemic, it's never gonna open again. There's a lot of precarity in the economy here, and it's begun to bounce back and we're seeing new restaurants and new activities coming up all the time, and another little perturbation is not really what we need, but so far it has not been a major effect on most people's lives. I feel bad for those that it has. Sorry about that. Scott D says, what are your thoughts regarding Eric Verlinde's ideas on gravity as an emergent force? And are they correct for our universe? Well, it's a little, yeah. It's a very good idea. But it's not unique to Eric. The idea of gravity as entropic force or an emergent force has been proposed in different ways by different people.

1:51:21.4 SC: What Eric did very, very well is sort of just make it actually kind of very tangible and physical and think about the Newton's laws rather than fancy general relativity kinds of things. But I will also give a shout out to Ted Jacobson at the University of Maryland who did related work earlier on what he calls the Einstein equation of state. And together those kinds of things from Verlinde and from Jacobson and some and from others, I think are super duper important and interesting. I think that we don't yet have a full understanding of what's going on, but their techniques, I've written papers about them, and I'm very much of the opinion that this is a very, very promising way to go forward for gravity, because the... I've said this before, but the short version is if you take general relativity and try to quantize it, you get stuck very quickly.

1:52:13.6 SC: It doesn't really seem to work. And this has led to ideas like string theory and loop quantum gravity and et cetera. But the other idea, and Tom Banks is another person who's pushed this line very consistently, is that you shouldn't start with a classical theory and quantize it. You should just have some truly quantum thing that gravity emerges out of. And so, yeah, Jacobson and Banks and Verlinde and others have all put on him on as another one who've all explored this idea. And I don't think that like we've quite yet crystallized on the right way to make it work. But I do think it's very promising. Yes. Edward Sackanger says, toward the end of your conversation with Matt Strassler, you mentioned that there are three languages to talk about gravity forces, curve space time, and a condensate of gravitons. Could you please explain to what extent these languages are equivalent?

1:53:08.2 SC: Is it possible, for example, to describe a black hole with gravitons in flat space time? Well it's possible, but it's not highly advisable. It's much, much better to describe a black hole as curve space time. So I think your point... The question is very well taken. The languages are not equivalent. Some are better than others. Some are more fundamental, more comprehensive than others. Gravity as curved space time is the most comprehensive way we have of talking about gravity right now. It works everywhere. General relativity works. Gravitons are the quantized excitation of space time. So if you have a background space time, which is either flat space or for that matter, a cosmological space time or a black hole or whatever, and then you do perturbations on top of that... Of the classical gravitational field and you quantize them, they look like gravitons.

1:54:00.9 SC: But those are only small perturbations. So those are specifically applicable to situations where the gravitational field is close to some known classical background and you're perturbing around it. The forces description, of course, is not really a separate description, it's just sort of a human scale kind of thing that begins to make sense in the Newtonian regime, which is not a weak field regime so much as everything is slowly moving, right? So you don't need to worry about relativity or anything like that. So I would say that the three languages are not quite equivalent to each other, but in the regime where you and I are sitting here in our rooms or in our cars or walking through the park or whatever, they all are equivalent to each other within that specific regime. Michael says, I know you're a 76ERs fan, but since it also seems you're a fan of basketball generally, I'm wondering what your thoughts are about Victor Wembanyama.

1:54:56.7 SC: I feel bad for the people out there who are not basketball fans. 'cause they miss out on the amazing person who is Victor Wembanyama. He's not the only amazing one, but he's a bit of a sensation in basketball circles these days. He's a rookie, he grew up in France and he now plays for the San Antonio Spurs. And what makes him amazing is he is a skinny 7'5 center. He's quite tall, like pretty darn tall when you think about it. I'm not sure if it's 7'4 or 7'5, but he's very tall. But that's not what makes him amazing. That's the point. Like we've had 7'5 people in the NBA before. He's just ultra talented. He can dribble, he can shoot, he can move, like very few people you've seen at that height or any other height.

1:55:45.5 SC: And it was interesting to see because early in the season, we're near the end of the regular season for the NBA right now. Earlier in the season, he did struggle a little bit he didn't like come in and dominate right away because the hype for Wembanyama or Wemby as we call him the the Wemby hype was quite intense. And he did not come in and set the world on fire, but now he is setting the world on fire. He adjusted right. He's a learner too, and that's what makes him scary. He's picking up skills. The only worry is if he gets injured, right? Because he just looks very fragile out there. There's another guy who's also a rookie this year, Chet Holmgren playing for Oklahoma State, who's almost as tall and approximately as skinny and also super duper talented. And it was Chet, who was the favorite for the rookie of the year in the first half of the year. But Wemby sort of has caught and passed him, but they're both just amazing. They're doing things that centers didn't do back in the day when I was growing up watching basketball. And it yeah, it bodes well.

1:56:51.0 SC: Well, let's just say that for the future of the NBA, there's a huge amount of not only talent coming in, but unique talent and really special talent to watch. And I think basketball is just beautiful when it's played well, the fact that it's a team game and the action doesn't stop. I do also say that this is very, very subjective and people can like different things. I get why people like baseball or soccer or hockey or American football or whatever, but these are all games where the scoring is rare. Right? Like, you have to wait a long time before someone scores. And in basketball, you score all the time. Like, half of the time. Half the time you have the ball. You're going to score, and you have the ball several times, a couple times a minute. So some people say, well, I don't like that they're just like scoring all the time. I like it to be more relevant, more impressive, more meaningful the moment they score. To me, what's great about basketball is that there's a rhythm that, yes, you score a lot of the time, but the fraction of times you score is not exactly constant. Right. And so you, it's a game of runs. Like, you're like, oh, up 10, oops, now we're down 15. What just happened there? And you got to keep it up. You can't relax. You can't wait for a miracle.

1:58:11.6 SC: You have to keep pressing. Every possession counts. The rhythm of the game is something that I think is very attractive in basketball. People don't like it for other, if people don't like it, that's completely fine. I'm not going to judge you one way or the other, but bringing the athleticism is wonderful because the court on which basketball is played is also smaller than any of those other games that we just talked about, which means that you can sort of zoom in and see the players. They're not wearing armor, they're not wearing helmets. You can see them. They're jumping and they're shooting and they're running and they're diving. And it's very beautiful when it all comes together. Anyway, I'm not very articulate at the moment, but it's good to see Wemby and Chet and the whole new generation coming in and rejuvenating the NBA. Donald Wilcox says, how would you describe spacetime in terms of gravitons? Well, I wouldn't. Gravitons are perturbative quantized perturbations of the gravitational metric tensor, right of the field that gives rise to gravity. So I guess the thing to say, is the following. There's spacetime, and then there's a bunch of fields on spacetime, right? The electromagnetic field, the Higgs field, the electron field, and what have you. One of those fields is the metric. And the metric is special because it tells you what the curvature, what the geometry of spacetime is.

1:59:46.5 SC: But even though it's special, it's still just a field on spacetime. At every point in spacetime, there is a value of the metric of the metric tensor, and the gravitons are the quantized excitations of that field. So it's, people are a little poetic and whatever, but gravitons are not spacetime. They are quantized excitations of one of the fields on spacetime, just like photons or quantized excitations of the electromagnetic field. And electrons are quantized excitations of the electron field, et cetera. So I wouldn't think about space time in terms of gravitons or even of the metric. Those are all things that live in spacetime. Igor Kopylov says, if Laplace's demon wanted to find an emergent, higher level theory of the system that it's simulating, e.g. Planets orbiting the sun, is there an efficient way for it to do that? On the one hand, we found those theories without being demons. On the other hand, considering every possible course, grading seems like a much bigger problem than even simulating everything at the lowest level. Well, gee, I wish I knew. That would be an awesome thing. In fact, I had a little proposal with a colleague of mine in the engineering school here at Hopkins to sort of see if we could automate that, see if we could write a computer program to find such emergent descriptions.

2:01:01.3 SC: We did not get accepted our proposal. Sorry about that. We didn't have enough time to put into polishing it, so it's sort of our fault, but. So, yeah, I don't know the answer to that. I mean, computer scientists are super interested in figuring this out, right? If you can find a good approximate description in terms of some compressed set of variables. And, by the way, it's not like they haven't tried. People have ways to try to do this. I just don't know how general those ways are. You know, principal component analyses, et cetera. They work in certain circumstances and don't work in other circumstances. So you're right. The number of possible coarse grainings is very, very, very big. How in the world can you and I find it? Well, it's because our observations are only sensitive to certain coarse grained observables and is a feature of the world in which we live. That not only is there a coarse grained theory in terms of some macroscopic variables, that works pretty well, but that those are the ones we can see, right? Those are the ones that our observations give us access to. We see positions and we measure velocities pretty straight forwardly. So, is that lucky, or is that just a feature of emerging core screening? I honestly don't know. These are the kinds of things I would much like to understand better, and I'm working to do so. So ask me again in 10 years from now. I'll tell you whether I've learned anything.

2:02:21.9 SC: Population Thinking says. I have a question about your discussion with Lara Buchak on risk and rationality. Can you give us some more intuition about the difference between a risk averse utility function and Buchak's risk avoidance? Well, I'm not the person to ask about this. These are details that require not only intuitive knowledge, but also knowledge of the technical lingo of rational choice theory. My impression is that utility functions, by definition, are not risk averse. That's not what they are. The idea of a utility function in rational choice theory is you have utilities for certain outcomes, and you have a way of calculating the utility under circumstances of uncertainty, which is you take the expectation value of the utility of the different outcomes. So, if you say, I'm going to flip a coin, and if it's heads, I give you a dollar. If it's tails, you give me a dollar, your expected value is zero, because there's 50-50 chance, and a 50% chance, I give you a dollar, 50% you give me a dollar, and you can say, well, I'm going to be risk averse, so I'm going to put really, really negative utility on losing the bet. Okay, that's fine. But the formula is the same, right? So, if you say, if it's heads, then you give me a dollar, and I'm happy. If it's tails, I give you a dollar.

2:03:53.3 SC: So I'm super duper unhappy. I'm tragically, devastated. So I'm going to give plus one utility for heads minus 1000 utility for tails. So that's just a different utility function. That's not a risk averse utility function. That's just a utility function that makes you very sad when bad things happen. Okay, I think that Lara's idea is that it's not about the utility you attach to bad things, so much as the probabilities that you have attached to them. In other words the, her suggestion would be that we can weight things not just by taking the expectation value of the probability times the utility, but that it is okay to sort of try to avoid to have a decision procedure which puts extra emphasis on avoiding bad outcomes, not just by giving them less utility, but over and above that, if that makes sense. I think I'm doing a bad job of explaining this, because that was a while since I did that podcast. But I think you have to distinguish between these two things, right? Between the very idea of attaching utilities to things which is perfectly compatible with conventional rational choice theory, and then the suggestion that even once you've attached your utility functions to different outcomes is the right thing to do to calculate their expectation value and try to maximize it. And Lara is trying to change that second step, not the first one.

2:05:23.6 SC: Owa says, why do matter antimatter collisions release an enormous amount of energy? If two inverse sound waves collide, the sound is nullified. Why does this not extend to particles, given that they are also waves? Yeah, because they are waves. But an antiparticle and a particle are not the same wave. They do not just constructively interfere or destructively interfere. They're waves in different kinds of things. So the idea of a particle, an antiparticle, is that they carry different kinds of charges, whatever those charges may be. In the simplest case of an electron and a positron, the electron carries a negative electric charge, the positron carries a positive electric charge. So an electron wave going up is not the same as an elect... So, I should say, I guess a positron wave is not the same as a flipped electron wave.

2:06:14.6 SC: It's a whole different kind of wave. But they're connected in the sense that they can come together, and since they carry opposite charges, they can convert into some other kind of field without any ill effects, right? Without charge conservation being violated or anything like that. The specific connection conceptually is that if you have a symmetry of your theory, and that symmetry gives rise to some conserved quantity like electric charge, then it will always give rise to one value and the opposite value. That's basically a feature in field theory. So if you have a relativistic quantum theory that has electrons in it, it will also necessarily have positrons, and those two things can, will be able to annihilate into photons, of course, but also gravitons or what have you.

2:07:10.0 SC: So the analogy with sound waves is just not very good. I should have just said that and left it at that. Richard Kashdan Kasdan says in the film Interstellar, the crew is walking around a planet whose gravity is so strong, that seven years on Earth pass for every hour they spend on the planet. So how come they walk around normally without being affected by this amazing gravity? If I'm remembering the movie correctly, it's not the gravity of the planet that is causing the time dilation, but the planet is orbiting a black hole. It's the black hole's gravity that is causing this time dilation. You have to be very careful. And I know that Kip Thorne was very careful. You might think, well, if the black hole's gravity is so amazing, why didn't it just rip them apart or something like that? But there is a difference between sort of the net pull towards something like a black hole and the tidal forces that you get in gravity. So the tidal forces are the things that literally lead to the tides when you have the moon going around the Earth. The reason why there are tides is because the part of the Earth that is closer to the moon feels a stronger gravitational force than the part of the earth that is opposite the moon. And so there's a distortion of the shape of the water on the earth.

2:08:26.0 SC: So that differential pull from place to place is what you need to worry about tearing you apart. So as long as the planet is relatively small compared to the size of the black hole, there might not be a big difference in the gravitational field from place to place, and therefore, no tidal forces to rip you apart. But the overall gravitational field can still be very big because of the black hole and therefore lead to this massive time dilation. Connor says, here's a question I had after listening to your latest episode with Claudia de Rham. As the universe expands, the furthest stuff we can see gets progressively red-shifted until we can no longer see it. Similarly, if you watch an object fall into a black hole, you'll see it get progressively red-shifted until it fades completely. Is there more to this similarity than just coincidence? Are there deeper connections between the cosmic horizon and an event horizon? Well, it depends what you mean by a deeper connection. They are both horizons, and this is a general feature of horizons, that there will be red-shifts associated with them. If you are maybe leaning toward saying that the cosmic horizon is an event horizon, no, it is not.

2:09:34.5 SC: And one very simple way you can see it is that the cosmic horizon depends on what observer you're talking about. Each observer will have a different cosmic horizon around them depending on where they are. Whereas the black hole's event horizon is a universal feature of spacetime, everyone agrees on what it is. The definitions are very similar, but subtly different. The black holes event horizon is the place where, when you pass it, you can never come back to infinitely far away, the outside world. The cosmic horizon is the place where, when you pass it, you can never come back to that observer whose horizon it was. Those are two slightly different things. Leo Behe says, I'm interested in the parallels between lossy data compression in computer science and scientific models of physical systems. In both cases, we're designing a sort of rule or algorithm that allows us to discard a large amount of information about a system while retaining the parts of the system that are meaningful to us. It seems that if we were Laplace's demon and we had all the information, we wouldn't need to describe it with physical laws. Do you consider this a potentially useful comparison, or is it too broad or vague to really be saying much?

2:10:41.1 SC: I don't think, so I'm not exactly sure what you're getting at, to be perfectly honest. But I wanted to clarify something, which is why I'm addressing the question. The whole point of Laplace's demon is that Laplace's demon is not told all information about a physical system at all points in space and time. Laplace's demon is told all information about a physical system at all points in space and one moment of time, and is also told the laws of physics. And Laplace's demon is supposed to have the computational capacity to use that information, the complete description of the system at one moment of time, to predict the system at all future moments of time, and retrodict the evolution at all previous moments of time. So if you're imagining a different kind of, let's call it a Humean demon, because Humean perspectives on the laws of physics say that the laws are not themselves part of the architecture of nature. They are merely convenient shorthand descriptions of what happens. So a Humean demon would have the entire thing that David Lewis called the Humean mosaic. That is to say, everything that happened in the whole history of the universe, past, present, and future, and not the dynamical laws. Okay, so the Humean demon would not need to describe the system with physical laws. Indeed. Now, that's a slight distinction between Laplace's demon and the Humean demon.

2:12:09.3 SC: Okay? But neither one of them has anything to do with lossy data compression. So there are both these hyper perfect kinds of demons that have perfect models of the physical system. No data is lost. I think that in scientific models or in computer science, we're a little bit more realistic because we're not a demon. And so we have incomplete information, and even the information we have is imperfect and has error bars and things like that. And we're asking questions about what we can nevertheless say about the system despite these limitations. And the answer is often quite a bit. Okay. But I do think that's a different kind of question than the demon question. Keep in mind, the whole idea of Laplace's demon is just a metaphor for the universe, so don't try to take too seriously the idea of a demon thinking and calculating and things like that. All Laplace was trying to say is that if you take Newtonian mechanics seriously, the universe is deterministic. That's all he was trying to say. And by the way, he was trying to say, but we're not. Therefore, we need probabilities. That's why the demon appears in his essay on probabilities. That's the whole point. Okay.

2:13:23.7 SC: Dave Grundgeiger says, in your recent solo podcast, the coming transition in how humanity lives, you expressed doubt that we could be in the same type of consciousness after uploading our minds into computer simulations. You said, if you take the information that is in your brain and encoded in some computer chip, you have removed its connection to your body, and what we think about as human beings is inextricably intertwined with their bodies. Why do you think that this objection won't be overcome by faithfully simulating the experience of having a body that doesn't seem different from simulating any other part of the world, which is already presumed in the premise. So I try not to say the same thing over and over again because I think it sounds boring. And it's possible that I didn't say the usual thing that I said in that particular podcast. I don't know, but I try very hard every time I talk about things like simulating the human brain into a computer to say, of course, in principle, you could simulate everything. You could simulate the body.

2:14:20.5 SC: You could simulate the whole world if you wanted to, but you don't have to. My, the point I always try to make is that if you simply take the information content, both the state of every neuron in your brain and the connectome, all the different possible connections between all the neurons in your brain, and you put that on a computer without trying to, in addition, simulate your whole body and your, I don't know, your endocrine system and your hormones and your breath and your blood circulate and all the different things that you are subject to.

2:14:57.9 SC: If you only did that, you would not capture what it is to be you. Of course, I'm a physicalist. I think you could do an atom by atom translation of your body into some other substrate, and that would be, that would have the equivalent dynamics to you. It would be just as real as you, etcetera. But that's enormous amount more work than just doing your brain. Even doing your brain is much more work than you might think it is. There's a lot of connections there in the connectome. So I just, It's there's just a point that you can't simply take what's in your brain, upload it and call it you anymore. That is the point Lemmy 101 says priority question. I'm using my priority question for a TV show recommendation, of all things. I absolutely loved your solo episode discussing time travel stories and was happy to hear you were a fan of time crimes, too.

2:15:47.7 SC: In describing the type of time travel stories that you like, which mirrors my own, my mind was screaming dark the entire time. Dark is a German show on Netflix and the most interesting, well executed, and complex time travel story I've ever seen. If you or anyone listening hasn't seen it, they should definitely give it a go. So, Lemmy, I appreciate you using your priority question. In fact, you know what, Lemmy? I'm going to not count that as your priority question, because, you know, just so everyone out there knows, I do read all the questions, even if I can't answer them the questions grow in number over time. I'm very sorry not to be able to answer all of them, but I do read all of them.

2:16:26.2 SC: And as I always say, my choice about which questions to answer are entirely governed by whether or not I think I have anything interesting to say about them. I did. In thinking about it, I realized that that wasn't entirely honest. It's also whether I would enjoy saying something interesting about them. And by interesting, I guess I mean interesting to me. So, anyway, if anyone else wants to make recommendations for TV shows or music or whatever, feel free to drop them in the AMA questions. You don't have to count it as a priority question. Lemmy I do know about Dark. I have not watched it. I very much appreciate the suggestion. Leroy says, your thoughts about designer babies in the recent solo podcast. Are there any specific issues or scenarios that you're particularly concerned about? As someone preoccupied with the history of disability, politics, medicine, and eugenics, I think this issue is likely more complex and ethically ambiguous than we imagine.

2:17:18.8 SC: Yeah, I completely agree that it is complex and ethically ambiguous that I'm going to be on board with my, I try to distinguish between things I think I have some clarity about and things that I have very little clue about. When it comes to designer babies, it is not that there are specific issues or scenarios that I'm particularly concerned about. The thing I think I have clarity on is it's going to happen. I have zero belief that 100 years from now we won't be flooded with designer babies. That's what I think is going to be true.

2:17:53.7 SC: I don't know what should happen. I don't know what kind of world that is going to lead to, et cetera, et cetera. I respect these questions. I think we should be addressing them more clearly than we are addressing them now. But I don't know what the answers to those questions are. That's my worry, that we're just going to kind of rush into it, pell-mell without thinking it through, and that's gonna lead to a mess. Maybe some good things will come out, maybe some bad things. I honestly don't know. Sometimes it's okay to say you don't know and I don't know. In this case, Ken Wolf says, both your recent solo episode on the transition in how humanity lives and the previous discussion with Sahar Heydari Fard on complexity, justice, and social dynamics prompted me to step back and think about the apparent implicit assumption that there is an agreed upon goal. I suppose preventing the extinction of our species is a base goal, though some might even question that. I would be skeptical about any general reference to human flourishing, since libertarians and various flavors of communitarians might have radically different and incompatible ideas about what that should look like besides just survival. Is there anything you think we can all agree on? Or will further divergent, heterogeneous social contracts be inevitable?

2:19:10.4 SC: Well, I don't think we need a common goal, necessarily, to exist and thrive next to each other. We do need to have some common, I don't know, rules, some common frameworks, some common guidelines right? About want wantingly killing each other and things like that. Like if your only goal is to kill me, then my goal is going to be to lock you up, not to try to let you have your way. But I think this is the miracle and challenge of democracy, right? Is that we need to allow for people to have divergent goals as long as they can find a common framework within which to live. I do think that people like John Rawls have tried very hard to specify exactly the difference between what we need to have in common in terms of our acceptance of others and so forth, and our values versus what we can diverge on. And in my impression is that those efforts have not been particularly convincing. I kind of worry about this as a general principle of democratic theory. I kind of worry that it is harder than we think to specify exactly what we need to have in common in order to be a functioning democracy. And in fact, that the extent to which democracy has functioned, which is quite imperfect through the years, has only been because we do have certain things in common and we sort of take them for granted.

2:20:47.7 SC: And as soon as those things stop being held in common, then democracy is not going to function as well. So I don't think that there's any, I don't think that there should be anything we can agree on other than a basic framework for living together. And I can't tell you exactly what that framework would involve, but it is something that we should try to get clear on, I would think, and hope. Anonymous says, have you found evidence for aliens or God? I would like to be anonymous. Okay, you can be anonymous. So no, I have not. And in fact, I will make you a further promise. If I do find evidence for aliens or God, I will let the world know via means of a solo podcast here on Mindscape. So keep listening. Actually, you know what? Even better, I will promise that I will make that known. The evidence for aliens or God. As a patrons only episode of the Mindscape podcast, only patreon supporters will get to hear what my evidence is. So I'm just saying, if you want to be in on what that evidence is going to be, in the case that it happens to come, sign up on patreon.com. Corey Riker says, I was thinking about symmetries and conservation laws the other day, and I became curious as to whether this relation could be thought of as stemming from orthogonalities orthogonality in Hilbert space.

2:22:07.3 SC: Disclaimer most of what I know about physics is from popular science books and podcasts. So I asked ChatGPT, and it applauded my insightfulness. It felt good, and I'm well aware that GPT, but I'm well aware that GPT is not perfect identifying true statements. So give it to me straight. Did I realize something true? Corey, you did not realize something true. Sorry about that. As I've mentioned before, Chat GPT, and other large language models right now at the current state of their programming are super bad at being factually accurate at advanced questions in physics. I mean, questions that are not typically asked, they're just not that good at those things. Even with like a couple of corrections, they can still sometimes be wrong. So it is a clever thought, but it turns out not to be true, because symmetries and conservation laws are of course related by Noether's theorem. That's the relationship. The relationship is when you have a continuous symmetry, you have a conservation law. Orthogonalities Orthogonality in Hilbert space are not, that I can see anyway related to that possibility, because orthogonalities Orthogonality in Hilbert space are all over the place, right?

2:23:15.6 SC: Hilbert space is the space of all possible quantum states. It is a vector space for many conceivable systems. It's infinite dimensional. It's possible that it's finite dimensional in reality. But still, the dimensionality is huge. And that means that almost all states are orthogonal to each other. If I have an empty space, or I have empty space plus one photon in it, those two states are orthogonal to each other, right? Any state with two different photons, one photon in two different places. So two different states, one photon each, but in different places, or N photons versus N plus one photons, etcetera. Any state like that, those two states are orthogonal. Almost all states are orthogonal to each other. Most interesting states that you might talk about. So I don't see what the connection would be. I mean, I could easily imagine a theory without any obvious symmetries or conservation laws, and yet there's still going to be states that are orthogonal in Hilbert space. So sorry about that, Corey. Don't believe everything the computer overlords over the loads tell you. Curious Dennis asks a priority question I have a nagging question about your biggest ideas in the universe.

2:24:22.7 SC: Given that religion has been virtually ubiquitous within all of our human societies, I think that meaning, or the search for meaning, might be as essential to the human organism as, let's say, vitamin C. One of the things that comes to mind when thinking of the night, when looking at the night Sky is. What is the meaning of all this? Don't you think that as a cosmologist like yourself, with a strong background in philosophy, you should include meaning as one of the biggest ideas in the universe? So if you read the biggest ideas in the universe, it's very clear, very explicit, not hidden, that I specifically mean physics ideas. Okay, maybe you can, sue me for false advertising. But that is the idea of the book. There are ideas about physics. There's plenty of ideas in biology that are not in the book. I do think that meaning is an important idea. So therefore, I recommend that you buy my book called the Big Picture, which is subtitled on the origin of, what is it? Yes, on the origin of life, meaning, and the universe itself. I talk about meaning in that book.

2:25:24.1 SC: I think it is a very big idea. Sorry, gang, I'm gonna have to start giving shorter answer to these questions. My voice has given out. I have a lot of questions I wanna answer it's Killing me, but I'm just not going to physically be able to do it. So, here we go. Nikola Ivanov says the ADS CFT correspondence doesn't reflect our observable universe, not only on the ADS side of the equation, but also on the CFT side. The standard models at QFT doesn't seem to be scale invariant and doesn't seem to exhibit conformal invariants. However, my impression is that scientific efforts seem to be focused on more, more on finding DS space correspondence to CFT, unless on replacing the CFT part of the equivalence with the QFT of the standard model.

2:26:06.7 SC: Is my impression correct? And if so, why is this the case? This is a little bit technical, I know for you folks out there, but remember, ADS CFT is a correspondence between a theory with gravity on the ADS side and a theory without gravity on the conformal field theory side. That's the answer to this question. So the question as to why we worry about having a better boundary theory than the CFT and ADS CFT is because we know we have gravity, the real world has gravity in it. So we want to find a dual that doesn't have gravity in it. That's the inspiration we get from ADS CFT, that we're going to be able to explain our gravitational theory with a non gravitational theory. So you can start with the standard model and you can remove gravity from it, but you're not going to get the real world out of that by taking some holographic dual. You already started with the real world minus gravity, different kind of thing. So that's what we're trying to do. We're trying to figure out what is the kind of dual description that doesn't have gravity in it, that could give rise to a more realistic phenomenological world with a positive cosmological constant like we live in? Chris Gunter says, could physical symmetries, like gauge symmetries, be akin to redundancies in information?

2:27:24.4 SC: Well, sometimes they are. In fact, gauge symmetries are precisely that. Gauge symmetries are redundancies. The they're like coordinate invariants. They're just saying there's an infinite number of ways to have coordinates, and they all give you the same physical predictions. That's what gauge theories do also, in a more subtle way, there are also things called global symmetries, which are not redundancies in information. They are just physical fields that could be different. But if they were different, the physics would still be the same, but there's still some degree of freedom associated with them that can have energy, et cetera. There's good reasons to believe there are no exact global symmetries in nature, but that's still a little bit conjectural. That's a quantum gravity kind of issue. Amy Ferguson says, do you often remember your dreams? Have any been especially memorable or led to a lucid dreaming experience? I usually don't remember my dreams. I forget them pretty quickly. I have them, and like many, often I wake up and go, whew, that was some weird dreams I had, and then I forget them 10 minutes later. Lucid dreaming. I got interested in lucid dreaming a long time ago, like when I was still in high school and I tried to make it happen, and I'm pretty sure I did make it happen once or twice, and then I lost interest in it.

2:28:37.9 SC: So I don't have any detailed experience with lucid dreaming, I'm afraid. No. Alan Lubel says, love your podcasts. It's great to have a healthy addiction. And I've listened to, again, to some of your early ones and noticed that you seem to laugh more often in them than in more recent ones. If I'm right about this, do you feel that since you were new to podcasts, the laughs were due to a little nervousness? Or rather, is it that in the present process of getting older, you become more serious? I don't know. It's a very short answer. I had never noticed that I laughed more often in the previous ones. I think that both of your suggested explanations are perfectly plausible. Let me put forward a third possible explanation, and then someone else can collect the data to empirically distinguish between them. Early on in the podcast, I put a lot of effort into trying to do all my podcasts, interviews in person. I thought it was like a little more human, better sound quality, things like that. And then the pandemic hit, and then it was impossible to do podcast interviews in person. And I kept doing them. I did them remotely. And what I noticed is that, in fact, I had better control over the audio quality if I did them remotely. Because when you do them in person, you have microphones in a room that you visit somebody in their lab or whatever, and you have no control over the environment.

2:30:00.2 SC: And if I just mail my guest a headset so they have earphones on and little microphone that is very, very close to their mouth, then number one, even if they move their head, the microphone moves with them. And number two, because the microphone is very close to their mouth, it's less likely to pick up noise from the outside world. I can actually get better audio quality through a remote interview than through a local one. And also I have access to a lot more people. And also, it's easier for me because I don't need to travel anywhere or whatever. So these days, almost all of my interviews have been done remotely. In fact, there was one podcast interview I did, I think, in the last year that was not remote. Let's see if you can guess which one it was, just by audio quality or by other aspects. I'll reveal it next month, if you remind me. And the point is that when you're interviewing someone, even though the audio quality is better, when I'm interviewing someone remotely, the spontaneity is not quite as good. I'm sure you've noticed if you've zoomed or skyped or whatever, with people, that for whatever reason, when someone is talking and you want to sort of butt in to interrupt them to ask a question or whatever, they often just ignore you. They just keep talking, right?

2:31:20.7 SC: And so therefore, you can't do that, and therefore you have to wait for them to stop talking, and then you have to go. So I think that that decreases the spontaneity of the conversation a little bit and maybe the laughter along with it. I don't know. That's just my hypothesis. Matthew Lounsberry says, priority question. While looking up to the sky, I've been tantalized with the thought that I can only see stars because those little photons have actually traveled all this way and entered and messed with some rhodopsin. Sorry, rhodopsin. I never get these biology words right in my eye, that it arrived with enough energy to change my being in some meaningful, if absolutely tiny, way. This feels to me significant since time for the photon slows to nothing and space shrinks before it. Is it fair to say that there is an actual point in the universe, the photons point of view, where distant Andromeda and the cells in my eye are connected at one place in time? Not really, no. Sorry not to be romantic about this, Matthew. The correct thing to say is that for that moment, there is a moment when the star or the galaxy that you're looking at was on your past light cone. I mean, of course there are many moments when the lifetime of the star is on your past light cone, but there is a particular moment that you are seeing right now that is on your past light cone.

2:32:44.1 SC: It would not be right to say that they're connected in one place and time, because there is definitely both space and time through which the photon travels. The proper time along the photon's trajectory is zero, but that doesn't mean that time doesn't pass or space doesn't exist. It's just that the stars give off a lot of photons, and we're fortunate enough to be able to see some of them. Kent Durham says your solo podcast indicated that we would reach a point to where firms would extract our wealth at maximum efficiency. A counterpoint might be that firms need to continue to compete against each other, and that this competition would lead to further enhancing the goods and services that we consume. That enhancement might preserve our standard of living ad infinitum. Do you have an interesting answer to this observation? Yeah, my answer is, come on. I mean, yes, I do know that there is something called supply and demand and competition and market forces and the free market. And in fact, I'm a fan of the market. I'm not against the market, but I also realize that it has downsides. As you can hear in podcasts that I've done with people like Suresh Naidu or Sam Bowles or whatever.

2:33:51.9 SC: The market is great and it is not perfect. Those are both possible. And so the point of the podcast was not that markets are bad and corporations earning profits is evil. That was not the point. The point is that the efficiency with which the corporations can earn profits can drive down our excess happiness in the world, because they're, of course, going to want to get all the profits they can. They want us to be just happy enough to buy their stuff. They don't need us to be any happier than that. And that's one of the ways in which market forces are not ideal for overall human happiness. So we need to compensate for them somehow, and I'm not sure how we're going to do it. Of course, there's also very obvious conditions where competition is not very effective, either monopoly power or monopsony power, as we talked about with Suresh, drive corporations away from perfect market outcomes. So there's still work to be done once you've realized the miracle of the invisible hand. Alex Thew says, I really enjoyed your discussion with Matt Strassler.

2:34:55.3 SC: There are many confusing ideas about physics that are probably owed to a limitation of language. For that matter, the idea of referring to a variety of fundamental things as particles have always confused me. Should we be thinking of these things as spheres, solids, or what? Well, yeah, that's perfectly fair, because, of course, they're not any of those things. They are excitations in quantum fields. They're not particle like, right? They're really not. Why do we call them particles? Because when you observe them, you don't see any finite size. There's no such thing as the size of the electron. If you try to probe electrons to smaller and smaller distances, they will just look smaller and smaller to you.

2:35:36.0 SC: So this is a feature of quantum mechanics that the world as it is is not the world as we see it. The world as we describe it when we're not looking at it is not the world that we measure when we do observations. So particle to a physicist just means it leaves a track in the detector, but a detector has some finite resolution etc. So you can always be more careful about it. I would not at all put too much ontological weight on thinking of the shape or the size of an elementary particle. If you really wanted to be more accurate you would think of fields and quantum mechanical measurements of those fields. Okay I'm gonna group three questions together. One is from Kyle Stevens. In your recent solo episode you outlined both optimistic and pessimistic visions for how humanity will live in the future. What can we average people do to increase the probability of the optimistic scenario coming to fruition? Nick Gaal says, I thoroughly enjoyed your solo podcast. You outlined optimistic and pessimistic scenarios, but what about a not better not worse just different scenario? All of our previous cultural transitions seem to better fit this just different characterization. Has any cultural era ever looked back and thought their era lived up to optimistic expectations or down to pessimistic ones?

2:36:53.4 SC: And then Philip Grant says, concluding your solo episode on the technological singularity, you rightly say we need to collectively decide to avoid the pessimistic possible outcomes and work for the more optimistic ones. How optimistic are you that humanity will find ways of making truly global collective decisions rather than stronger nations imposing their interest on weaker ones? So the idea behind me offering the optimistic and pessimistic scenarios was twofold. One is reflecting true uncertainty in what I think might happen. I try to be honest about what things I am confident about and what things I am not. And I think that maybe this is not the way to the bestseller lists and to punditdom. But I think that in this case, I think that dramatic changes are coming. And I'm not sure whether the positive aspects of those changes will outweigh or not the negative aspects of those changes. So sketching out the extremes I think is a useful exercise. Not because I'm secretly predicting one or the other, but because it's sort of is important to keep in mind the possible upsides and downsides. So probably to answer Nick's question, we will end up somewhere in the middle of them. That's what I would guess. But there's a huge amount of room in between the middle of them. And for that matter, there is very plausibly positive, outcomes and negative outcomes that I didn't think of, right?

2:38:20.3 SC: That would make things even better or worse. So it's again, as I tried to say in the podcast, it's an exercise in thinking through the possibilities. It's not an attempt to really make predictions. For what Philip says, how optimistic are you? I mean, I'm not super optimistic. So if I needed to guess, I don't wanna guess, I don't wanna do this. But if I needed to guess, I would think there are gonna be many true, good, positive outcomes of these technological changes. And they're gonna be 100% exploited by more powerful interests to squeeze the less powerful ones. That's not much of a dramatic claim, right? That's typically what happens with technology. Both of those things happen. So that's a somewhere in the middle kind of thing. Kyle's question on what can people do to increase the probability of the optimistic scenario? I think, again, I don't exactly know, but I certainly think that being well aware of what is going on is important. I think that recognizing methods of effective political change and being politically active and engaged and not being one of these people who sits back and goes, politics is all corrupt, both parties are the same, etc., etc.

2:39:36.6 SC: I think that there are influences that bubble up from people to politicians and you can put pressure on them by voting and talking and trying to make the world a better place, talking to each other, and also being open-minded, not deciding ahead of time what is going to happen and just being stuck there, but being willing to change your beliefs as things happen. You know, this is all good general purpose advice, not just for this particular question. Reid Atherton says, I'm intrigued when you said that modern levels of productivity imply that we can meet the basic needs of all people, but I'm curious what the feasible method for actually doing this might be, leaving behind a spherical cow understanding. For instance, centralized economic control has been worse at incorporating all the necessary information into managing surpluses and shortages compared to market pricing. That's very true. I agree that centralized economic control has been kind of disastrous. So my idea is a little bit radical, but my idea is to let the market work, let some people earn more money. And then the people who earn a lot of money, the government should take some of that money and it should take more and more of your money as a fraction of what you earn, the more money you earn. Once you're past a certain level of basic living in the world, you need less and less of that excess income.

2:40:58.3 SC: You still wanna keep some of it to give you motivation to keep going. But nevertheless, I think that the society should be able to take a good fraction of it since after all, once you get to these very high levels of income, as we discussed in the podcast, you're not earning it by the sweat of your brow. You're earning it by some system that gives you accumulated multiplicative ways of earning. And then you can take that money that society has extracted from you and give it back to those who are not doing well. I don't know if anyone will go along with this, but that is my radical economic plan. Ilya Lavov says, you often close your pop-sci explanations with a line akin to, if you wanna understand more, you need to look at the equations. I understand where this comes from, yet I can't help but make a mental connection to a particularly difficult course in macroeconomics I had in my undergrad studies. That course was so difficult precisely because the equation, the professors made us go beyond solving the equations. The equations were always the easiest part by far. The difficult parts were to give a causal interpretation and to give a graphic interpretation in term, to graphically show the movement of fundamental economic curves.

2:42:11.1 SC: Is there a similar risk in physics that focusing solely on the equations would strip away some of the understanding? And what other modes of understanding beyond the equations are similarly helpful to achieve a deeper understanding in the physics studies? So I don't really think it's empirically true that I often close my pop-sci explanations with if you wanna understand more you need to look at the equations. Sometimes I do but it's a very specific question to what is trying to be addressed. For some issues there are very visualizable, tangible analogy or metaphorical based ways of conveying the information. For others there's not because something is going on that is outside of our usual understanding and the math is perfectly clear about it and our everyday experience is not. A classic example is energy conservation in the many worlds interpretation of quantum mechanics. There's an example where if you understand the equations at all you have zero questions like perfectly obvious what's going on but if all you understand is like the picture in your mind that, oh, there's a whole nother world out there, where did the energy come from. It can seem very confusing to you and I can try and I have tried to you know convey some feeling of what the equations are trying to tell you but it's not as good as just actually looking at the equations, that's just true.

2:43:32.7 SC: But of course that is completely different from saying that all you have to do is understand the equations. There's levels of understanding anything any good physics course would not be about just understanding the symbols and equations or even just about solving them. Of course you wanna understand the kind of intuitive behavior of the equations you wanna understand their limits you wanna understand approximations you wanna understand perturbations and stability you wanna understand you know maxima and minima and what the you know, general behavior of the equations is under different circumstances and it's all understanding physics that matters. The equations are just the precise statement of those physics they're not either more or less than that. Kevin James says my understanding is that it takes negative gravitational energy to create a wormhole it appears that dark energy behaves sort of like negative gravitational energy. I know you'll say yes it's possible but I'm asking anyway if we could hypothetically harness dark energy as best we know about it how could we create wormholes? Well Kevin you'll be happy to learn I'm gonna say no. I'm not, not that it's impossible to make wormholes that might be very well possible but harnessing dark energy will not help. Indeed and this is just coincidental in how I didn't plan this out but here's a classic example of where the equations make the answer perfectly clear and the words kind of muddle it a little bit.

2:45:02.5 SC: It is true that the dark energy we think is accelerating our universe has a vaguely anti-gravitational feeling about it right? It's pushing away things faster. Things are moving away from us. That's kind of anti the usual behavior of gravity, but in fact if you look at what the equations are telling you the energy density of dark energy that you need to explain the universe is a positive number not a negative number. There's different ways to reconcile the fact that it's a positive energy with the fact that things are accelerating.

2:45:33.5 SC: The most common way is to say that in addition to the energy density there is also a negative pressure and the net effect is driven not by the energy density by itself but by the energy density plus three times the pressure and if the pressure is negative and equal magnitude to the energy density p rho plus 3p is a negative number rather than a positive number. That is literally the most common way of explaining this and is completely opaque and you shouldn't feel bad if you don't understand it but there is pressure in addition to energy density that's a feature of relativity. My way of explaining it is to say that the point is just that the energy density is positive but it doesn't go away it doesn't fade away right, that's the real characteristic of that you need for dark energy that it's more or less constant as the universe expands and what that means according to general relativity is that the curvature of space-time cannot go to zero because there is always some persistent non-zero energy density in it.

2:46:38.4 SC: And in expanding universe a homogeneous and isotropic universe like we seem to live in on very large scales there are two sources of curvature of space-time. One is the curvature of space all by itself and the other is the expansion of space, the expansion rate the Hubble parameter okay. These two sources of space-time curvature and guess what, one of them is zero as far as we know. The universe does not seem to be curved in space as far as we can measure. So the curvature of space-time is just numerically it's proportional to the Hubble constant squared. So rho being constant the energy density being positive and constant tells you that the Hubble parameter will approach a positive constant. And the Hubble parameter, what does it do? It tells you if you look at a galaxy, the velocity that you see for that galaxy is the Hubble parameter times its distance, right? V equals h times d. D is the distance, v is the velocity. So if h is a constant, which it hasn't been through the history of the universe, it's been decreasing because the universe is becoming less and less dense, there's less and less space-time curvature, but as we are getting taken over by dark energy, h goes to be a constant, the distance to any one galaxy increases with time.

2:47:58.6 SC: So the velocity is h times the distance, constant times increasing number, that's gonna be an increasing number. That's the situation we're in right now, that's why the universe seems to be accelerating, nothing to do with anti-gravity at all. To make a wormhole you literally need negative energy, which is the opposite of the dark energy as we have it right now. Henry Jacobs says, in the bonus reflection video for Sahar Haidari Fard, you were struck with the notion of cultural revolutions as critical phenomena, where an idea can spread like wildfire if the network is scale-free. This reminds me of stuff in the air in the mid '90s, often at the Santa Fe Institute, related to the sandpile game. Not a specific question here, but do you have any thoughts on the sandpile game and the research around it?

2:48:43.0 SC: I get the impression that it fizzled, but maybe not. It fizzled a little bit, I would say, so what Henry is referring to is the idea promoted by Peer Bak among other people, that if you take a pile of sand and you dribble sand on top of it continuously from the very top, it will reach a more or less steady state configuration because it wants to get steeper and steeper, but the steeper it is, the more likely it is that there's a little avalanche where some of the sand falls down. So you can do statistics, and at least in some regimes, there is a critical angle for the sandpile to be at, which is what you reach at this steady state, and then the spectrum, which is to say the set of all possible avalanches you could observe looks scale-free.

2:49:27.8 SC: It looks like a power law, as we were talking about before. So there's many little avalanches and a few bigger ones, and they obey this sort of scale-free behavior. And this is the origin of, or this is an example of what was thought of as self-organized criticality. Per Bak even wrote a very nice popular book about it. I believe, and I'm not a super expert here, I believe the current consensus is that it was a little overhyped, the idea of self-organized criticality. It's less, you know, again, there's many ways for systems to become scale-free. There's not, as far as I know, a single unified explanation for all of them. So the idea that self-organized criticality, similar to what happens in sandpile dynamics, would be the underlying universal explanation has not panned out. That's not to say that there aren't some examples where it is perfectly applicable. I think it's not just not quite as universal as people thought. So as often happens, it's a good idea, but the hype cycle gets ahead of itself a little bit and eventually there's certain aspects that remain valuable and others didn't quite pan out.

2:50:36.9 SC: Okay, I'm going to group together two questions. One is by Sandro Stuckey. In the episode with Matt Strassler, you both agree that particles in quantum field theory are not really point particles, but modes of excitations in a quantum field. But Feynman diagrams look like they describe the interactions of point particles. What am I missing? And Claudio says, the physics of an incandescent light bulb seems pretty straightforward. Electrons running on a thin tungsten wire experience resistance, generating photons and heat according to simple equations. But how does it work in terms of quantum field theory? Can you explain how, in the space occupied by the wire, excitations in the electron field turn into excitations of the electromagnetic field, but only under precise conditions? Why are properties current, etc.? I know these seem like very different questions, these two, but my answer to them is going to be the same, which is that there is a way of describing what happens in quantum field theory, which is the quantum theory of fields. You have a wave function for all the fields. The fields are, you're sort of providing a basis for your wave function, etc., etc. But by studying the quantum field theory, and I will get into this in my book, if you wanna read my book as well as Matt's book, I think he gets into it also, the excitations of the field show up to us as particles.

2:51:53.7 SC: So particles can be a perfectly valid way of talking about what quantum field theories do in certain regimes. As I talk about in the book, inside a proton is not a good place to think about particles. So people always say there are three quarks in a proton, but a proton is not really made up of three quarks. That's not really an accurate picture. It's made of excitations in quantum fields with the quantum numbers of three quarks, but that's a more subtle statement. It is true, as Sandro says, that Feynman diagrams look like they describe the interactions of point particles, and there's sort of two things to say about that. One is, yeah, they do, and that's a pretty good approximation to what's going on. If you dig into what the Feynman diagrams are actually telling physicists, they're not supposed, the Feynman diagrams are not supposed to represent particles at a certain location in space.

2:52:50.5 SC: If they did, it would be weird. Like, what are the, what's the probability those two particles would hit at exactly the right point to make that Feynman diagram? The particles, the lines in the Feynman diagrams actually represent waves. They represent plane waves of constant momentum spread out through all space. So they're attached to momentum, not to position. That's a little bit of a subtlety, but the point is at the end of the day, the predictions you make can be reinterpreted as particles moving with certain trajectories, and that's what you need to do. Likewise, for Claudio's question, the way to explain how excitations, electron field, etc. Turn into excitations, electromagnetic field in a wire is first just explain why excitations in electron field look like electrons, and excitations in the electromagnetic field look like photons, and then use ordinary electromagnetism. That's a perfectly legitimate thing to do. Edward Crump says, if string theory is correct in some form, could the strings purported to constitute the fabric of space pass through the nucleus of an atom and even between two quarks? Yeah, in fact, the strings are particles, right? I don't think it's accurate to say the strings constitute the fabric of space. I'm not quite sure exactly what that means. Again, there's a difference, as we said just a little while ago, between gravitons and space-time. Strings are more like the gravitons.

2:54:13.4 SC: One excitation mode of a string is a graviton. A different one is an electron. A different one is a quark, etc. That's what the strings really are. Matthew Atkins says, how important is the cosmological principle to modern cosmology? Do we have good reason to believe it or is it just a convenient assumption? So the cosmological principle, for those of you who don't know, is the idea that the universe is more or less the same everywhere, right? Like, obviously, it's different here on the surface of the Earth than in interstellar space, but if you average over large distances, the number of galaxies, etc., is more or less the same. This was a very helpful, it's not a principle, let's put it that way. It's not any reason that it needs to be true, like a real principle should have. It was a very helpful assumption when people started doing cosmology. They didn't have a lot of data. We're talking about the 19-teens, right? We didn't even know galaxies were far away. We thought the whole world was the Milky Way galaxy. So just assume the universe is more or less the same everywhere as a convenient starting point, then you check it against the data. That's what you always do. And these days, we have the data, right?

2:55:22.0 SC: We have much, more accurate data about the universe on large scales, largely from the cosmic microwave background, but also from maps of galaxies throughout the universe. So who cares about the cosmological principle? We know where the galaxies are. We know the isotropy of the universe from the microwave background. That's all we need to make progress. Rad Antonov says, in your view, does the Everettian interpretation of quantum mechanics resolve apparent fine-tuning problems such as CP symmetry and QCD and the values of certain constants by allowing for the existence of an infinite number of combinations of coupling constants? No, for two reasons. Number one, it doesn't allow for the existence of an infinite number of combinations of coupling constants. At least in the usual way of thinking about Everettian quantum mechanics, the coupling constants are the same in every branch of the wave function. It's the same underlying laws of physics, including the constants that appear in whatever quantum field theory you're using to run your wave function and you're shorting your equation. But the other is, even if you could have an infinite number of values, there's still no reason for the value of, let's say, the CP violating parameter in QCD to be very, very small in our universe.

2:56:41.6 SC: This is something called the strong CP problem, for those of you who are not familiar. There is a parameter that, as far as we know, could exist in quantum chromodynamics that would be easily experimentally detectable. It would violate the symmetry called CP. It's a number between zero and pi and in principle, once again, but then you measure it and the answer is it's less than 10 to the minus 10. So it could be exactly zero as far as we know, but we don't know why it is so much smaller than order one. There are theories like the axion was invented to exactly explain this symmetry. The Peccei-Quinn symmetry was invented and then Frank Wilczek and Steven Weinberg pointed out that it leads to a particle called the axion, but many worlds have no help here. You need some dynamics to make that happen. Paul Hess says, we visited Johns Hopkins yesterday as part of my son's college search and stopped by the Paper Moon Diner based on your recent mention of it. He says Hopkins is elevated on his list in part because your role there is a sign of openness towards interdisciplinary thinking and he likes the annual lecture series you've mentioned helping to organize there. So take this as a small sign of the positive feedback loop that science communication and outreach efforts can have. He'll be applying to schools with a focus on classics but finds appeal in philosophy, physics, and math so is focusing on medium plus schools that have strengths in all these areas.

2:58:05.0 SC: This is not a question, just a thank you and I'm equally happy if you comment or don't comment on this among the flood of actual questions. I just wanted to thank you Paul for letting me know. I'm glad that your son has such good taste whether he ends up coming to Hopkins or not. I think that his thought process sounds good to me. You know there's a lot of schools out there. You can't rank schools really. People always try to but individual students are going to find their own success at different schools depending on what it is they want. So it's important as a prospective student, not just to think about where schools are ranked but what they're good at, what they're less good at, what's important to them, what the student body is like, what the professors are like, what the size is like, what the location is, like a million different things to get your particular optimal solution there. So good luck to him and to everyone else in that situation. Yohannathan Peretz says if we find through learning about quantum gravity that the space-time curvature doesn't go to infinity inside a black hole, would that do any violence to the statement that no light can escape it? No. It would do no violence to that whatsoever. Because the statement that no light can escape it is a statement about the event horizon which is typically very far away from where the space-time curvature goes to infinity at the singularity.

2:59:19.5 SC: Very far depends on how you're measuring but they're two unrelated things. I mean they're related because they're both involved with black holes but there's no direct connection there. Qubit says if we are faced with a very hard decision, do you think the resulting outcome corresponds to two branches with roughly equal weight or is the overwhelming weight still concentrated on one of the two branches? The latter seems to speak against your statements about many worlds being comforting because you know yourself to have chosen differently in another universe. So I don't know what I've said in every all these many many hours of speaking that I've done but I certainly never meant to say that many worlds is comforting because you know yourself to have chosen differently in another universe. That's literally the opposite of what I think, feel and have often said. I will sometimes joke about it in the context of you know the universe splitter app or whatever but I said many many times the people in other branches of the universe are not you anymore. They share a past you, an ancestor you, but they're not you anymore so you shouldn't take any comfort in them at all. But for the question I think it is the latter. I think the overwhelming weight is generally concentrated on one of the two branches. Why?

3:00:33.3 SC: Because as I said before in the brain dynamics are pretty classical. The brain is a very decoherent system. There's not a situation where your neurons are in a superposition for a very long time and can collapse one way or the other with substantial probability. At least as far as I know. I think that's the way to bet but there's a lot we don't know yet about the brain. William Briggs says I'm a third-year medical student considering a career in academic pathology. What advice do you have to consider academia? Should I try to find my area of research and set myself up to be successful? How should I try to find my area of research and set myself up to be successful? I am not gonna be able to give you very good advice William. I'm glad you're thinking about these questions. These are important questions but everyone's path here is different. There's a million different ways to succeed. There's no one right or wrong way. And so It very much depends on the combination, the interaction I would even say, between your specific interests and skills and the rest of the world. You know, there are things you are good at, there are things you love doing and are interested in, and there are things that the rest of the world cares about.

3:01:46.4 SC: I always encourage people to look at the intersection of those three things. And then there's a sub-dominant contribution which is if you have an intersection of things you're interested in and you're good at and the world cares about, try to do the high-impact ones, right? Try not to just be impressed with doing research for the sake of doing research, but do the best possible research you can. Try to do research that makes an impact that really adds to our knowledge of the world in some way. How to actually balance all those factors and make it happen, that's gonna be up to you, I'm afraid. Sorry about that. Orin Cummins says, you know, now I'm self-conscious every time I say sorry about that. I'm ruined now after this question that opened this AMA. Okay. Orin says, does two plus two equals four in every branch of the multiverse and what does that tell us about the underlying structure of many worlds? Yes, two plus two equals four in every branch of the multiverse, at least given the right axiomatic system for arithmetic. You know, you need to define what you mean by two and plus and equals and four and different systems will have like in mod base three arithmetic, it's not true. But taken, I'm taking your question to mean in ordinary natural numbers under ordinary notions of arithmetic, then that follows from math, not from physics.

3:03:10.7 SC: And so it's gonna be true in every branch of the multiverse. And that tells us nothing about the underlying structure of many worlds other than the fact that logic is independent of physics which we knew already. David Wright says there's an ongoing debate regarding the ethics of generative AI producing images music and literature without crediting the original human contributors. My intuition is that the published material represents a kind of collective unconscious that AI draws on to create its outputs. This could lead to a feedback loop where AI shapes our collective experience and influences our future. Over time AI could produce documentaries literature and cultural beliefs that alter our understanding of history. Is this a serious risk to our civilization? Well it could be. It could be a serious risk if we just let it happen. As I'm recording this, there was just a kerfuffle because on X, the website formerly known as Twitter, you know there is a new AI agent called Grok which feeds up stories to people and it completely makes up stories. It made up a story about, I forget, it was like Israel invading Iran or something like that. There were other stories that had zero connection to things that really happened. So there's absolutely a worry that if you don't control the AI and what it's gonna do, then it's going to pass off true things as or false things as true things, right?

3:04:40.6 SC: I'm not sure about the specific sort of cultural unconscious aspects that you're pointing at. You know, one of the reasons why it's non-trivial to make an AI agent is because you're training it on a whole bunch of proprietary stuff, right? You're training it on copyrighted books and issues of newspapers and magazines and things, but then you're trying to make it produce outputs that are not obviously plagiarized, right? And many people have shown that you can easily trick it into basically plagiarizing, right? To making, If you prompt it with a certain kind of request for a cartoon, it will draw a Bugs Bunny even though it's not supposed to, you know? So I don't know how to stop this from happening or what a big risk it is. I think these are all aspects of the great unknown that we're gonna blunder into.

3:05:34.9 SC: The thing that I do know is that we should think about this carefully and we're not gonna. We're just gonna do it and there's gonna be some bad ramifications before we figure out how to clean things up. Ken says, what is the answer to Elizabeth's question to Descartes? Given that the soul of a human being is only a thinking substance, how can it affect the bodily spirits in order to bring about voluntary actions? Right, the answer is that the soul of a human being is not only a thinking substance. Descartes was a dualist. He thought that the mind was separate from the body. The right answer is that the mind is an emergent phenomenon that depends on the body and supervenes on the body, in particular what's going on in the brain and the nervous system and so forth.

3:06:18.0 SC: That answers the question perfectly. They're on a closed causal system. Kyle Cabasares says, at what point did you subscribe and hit the like button to the many worlds interpretation of quantum mechanics? I'm interested in knowing if it was before, during, or after your first formal introduction to quantum mechanics in the classroom. Oh, it was well after. Before I had a class on quantum mechanics, I didn't really know much about it. And When I did take quantum mechanics, it wasn't an especially inspiring course. It involved a lot of solving differential equations with boundary value problems and square wells and doing the WKB approximations to get tunneling times and things like that. That's what you learn when you actually take quantum mechanics as an undergraduate. And I didn't really think that deeply about quantum mechanics. I started thinking slightly more deeply, still not super deeply, when I was in grad school. And Even though I never actually did research on this at the time, one of my professors, Sidney Coleman, who taught me quantum field theory, was the author of a very influential idea. For a little while, it was influential. That kind of died out, although people still remember it, about Euclidean quantum gravity in quantum cosmology and how that could solve the cosmological constant problem.

3:07:33.3 SC: And I thought this was intrinsically interesting and I read up on it. And I even was a co-author on a review article on the cosmological constant where I explained it as best I could. I tried to learn about it carefully. And Quantum cosmology generally, the attempt to apply quantum mechanics to the universe, makes sense in an Everettian or many-worlds perspective in a way that it just doesn't very much in other perspectives. Indeed, as I've often remarked, Everett was assigned the thesis project from John Wheeler, his advisor, of quantizing gravity. And He thought about that as quantizing the whole universe and he soon realized, well, if I'm an observer, or rather, let's put it this way, if quantum mechanics requires that there be an external observer measuring the system, how could that possibly apply to the whole universe? And That's why he invented many worlds. So it's not a necessary connection. You can do quantum cosmology without it, but they fit together very, very nicely. And so almost everyone who thinks about quantum cosmology is an Everettian, explicitly or otherwise. And I read about that and thought about it and it made perfect sense to me. David Maxwell says, I love your solo episodes. They're always on engaging topics and elucidate the topic in a way few other sources do, even when you're not the subject matter expert.

3:08:52.8 SC: Your latest one on the coming transition was brilliant and timely. So why don't you do more? I could see reasons including additional effort, having a finite number of things to opine on, solo being less popular with occasional listeners or just that you started this to interview others, not give monologues, but I adore them and suspect I'm not alone. Thank you very much, David, for the encouraging words. There's a lot of different reasons why I don't do more. I thought about doing more. I did once a while back say, should I do more? And the consensus was no, not because they don't like them. In fact, the solo episodes are among the most popular ones in terms of downloads and whatever. But there's a frequency that kind of works. They work as occasional things. If ice cream is your favorite dish, you still don't wanna have ice cream for breakfast, lunch, and dinner. You wanna have it as an occasional treat. And I think a similar thing goes for the solo episodes. I like doing ones that are a mix of things that I am an expert on and not an expert on with appropriate humility, etc. But I do want to, there's fuel for all these discussions that come from the non-solo, from the interview episodes, where I get someone who is an expert on something that I am not, bring in new material to chew over, etc.

3:10:12.1 SC: So I think it's all part of an ecosystem. They're a little bit easier to do overall. I mean, at least maybe it's just more fun for me 'cause I'm only thinking in my brain rather than somebody else. So I could do more, but you know, probably the quality would go down if I tried to do too many more than I already do. So I appreciate the support, but I think there's good reasons to do it at about the current level of appearance. Final question comes from C. Handley. I'm gonna give a disappointing answer to C. Handley's question. Sorry about that. The question is, I hope Ariel and Caliban can help me with this marital issue. When my wife and I got married, we made up some silly little vows to go along with the more serious ones. At the time, she had one cat, Gatsby, that I understood would be part of the family. I like cats, but not as much as my wife. One of her vows was, I understand, we will never have more than one cat. We now have three, Stella, Penny, and Pete. I argue that since the vow was broken at two different times, I have two vow breaks in my back pocket for future use. She argues that a single vow was broken, period, so that's just one allowable vow break. Can physics provide some insight into who is right here? Also, am I warranted to have a relatively high credence that when I break my vows, she will understand?

3:11:36.6 SC: The final question about understanding, I cannot really provide any insight on here, but I will opine more from the perspective of a legal judgment here rather than a physics judgment. Physics is of no use whatsoever in this question, but given that it is a vow we are talking about here, we are taught both by legal history and from stories about genies granting wishes and so forth, that one has to be very careful, also stories about signing deals with the devil and selling one's soul. One has to be careful about the precise wording of vows. That's something that goes along with the concept of a vow, that you have to word it in such a way that it can't be wriggled out of.

3:12:21.2 SC: Now by your own testimony, the vow was "I understand that we will never have more than one cat. The vow was not, I understand that we will never get another cat in addition to the one we have. That was not the vow, right? So the vow was, we will never have more than one cat. So that's a yes no question. You have more than one cat or you don't. Having three cats is having more than one cat just like having two cats is having more than one cat. It's not extra having more than one cat, it just isn't. So I would claim, I would rule, I think that I have authority here that the vow was broken but it was only broken once. So C. Handley, you have one vow break in your future that should be used to even out. Of course this is assuming that all the vows are created equal. Maybe they aren't. I don't wanna get into that. That's for an appeals court to decide. So that's my ruling. Sorry it's not the one you were looking for but I think that Stella, Penny and Pete sound like pretty awesome cats. You should be happy that vow got broken. Thanks everyone. We made it through this far. I'm gonna talk to you next week. Bye bye.

2 thoughts on “AMA | April 2024”

  1. Good explanation of why particles in quantum field theory (QFT) are not really point particles, but modes of excitations in a quantum field. Yet particles can be a perfectly valid way of talking about what quantum field theories do in certain regimes, for example as a way to explain why excitations in electron fields look like electrons, and excitations in the electromagnetic field look like photons, and then use ordinary electromagnetism. That’s a perfectly legitimate thing to do.

    I recommend the short book ‘QED The Strange Theory of Light and Matter’ (1988), Richard P. Feynman to anyone interested in physics at an elementary level.

  2. Sorry about that… Get Smart TV series 1965-1970 major catch-phrase from the lead character

Leave a Comment

Your email address will not be published. Required fields are marked *

Scroll to Top